You are on page 1of 65

 0484-3190310

 9446331522
9446331522
TARGET PRELIMS 2017

1. The Base erosion and profit shifting (BEPS) action plan has been developed by OECD to help countries
to tackle tax avoidance by Multi-national companies. Which of the following statements is/are correct
in this context?
1. The BEPS package provides 15 Actions that equip governments with the domestic and
international instruments needed to tackle BEPS.
2. The OECD BEPS action plan is legally binding on the participating economies and non-complying
countries will be subjected to economic sanctions from OECD.
3. Action 6 of the 15 Actions recommends stringent measures to prevent treaty abuse and hence, is
significant for India.
Select the correct answer using the code given below.
(a) 1 and 2 only (b) 1 and 3 only (c) 2 only (d) 1, 2 and 3
Answer (b)
Explanation
Developed in the context of the OECD/G20 BEPS Project, the 15 actions set out below equip
governments with domestic and international instruments to address tax avoidance, ensuring that
profits are taxed where economic activities generating the profits are performed and where value is
created.
They are soft law legal instruments. They are not legally binding but there is an expectation that
they will be implemented accordingly by countries that are part of the consensus. Action 6 of BEPS
recognises that treaty abuse, and in particular treaty shopping, is a critical BEPS concern. It
recommends stringent measures to prevent treaty abuse and hence, would help India, which has
been a victim of treaty abuse by Multinational corporations to curb treaty abuse in future.

2. India has been amending Double Taxation Avoidance Agreements with many nations recently. What
are the likely advantages India perceive from this move?
1. Tackle treaty abuse.
2. Check round-tripping of funds to India.
3. Curb revenue loss.
4. Ensure maximum double non-taxation.
5. Streamline the flow of investment.
Select the correct answer using the code given below.
(a) 1, 2, 3 and 5 only (b) 2, 3 and 4 only (c) 1, 4 and 5 only (d) 1, 2, 3, 4 and 5
Answer (a)
Explanation
The amendment of double taxation agreements with the nations would help India to tackle the long
pending issues of treaty abuse and round tripping of funds attributed to the India-Mauritius treaty,
curb revenue loss, prevent double non-taxation, streamline the flow of investment and stimulate
the flow of exchange of information between India and Mauritius. Improve transparency in tax
matters and will help curb tax evasion and tax avoidance.it would prevent double non-taxation instead
of ensuring it.

3. ‘Project SAKSHAM’, a new Indirect Tax Network of the Central Board of Excise and Customs (CBEC)
will help in
1. Implementation of Goods and Services Tax (GST).
2. Extension of the Indian Customs Single Window Interface for Facilitating Trade (SWIFT).
3. Implementation of taxpayer-friendly initiatives under Digital India.
4. Improving Ease of Doing Business with regard to Central Board of Excise and Customs.
Select the correct answer using the code given below.
(a) 1 and 2 (b) 2, 3 and 4 (c) 1, 3 and 4 (d) 1, 2, 3 and 4


Answer (d)
Explanation
Project Saksham, is a new indirect tax network of the Central Board of Excise and Customs (CBEC).
The total cost of the project is estimated to be Rs 2256 crore, which will be incurred over a period of
seven years.
Project Saksham will help in the implementation of Goods and Services Tax (GST). It will also help in
the extension of the Indian Customs Single Window Interface for Facilitating Trade (SWIFT) and other
taxpayer-friendly initiatives under Digital India and Ease of Doing Business of CBEC. The introduction
of GST will result in a several-fold increase in the number of taxpayers and resultant document load
on the system. CBEC’s current IT system cannot cater to the increased load under GST without an
immediate upgrade of its IT Infrastructure, which is aimed through the project. It will also help in
other taxpayer-friendly initiatives under Digital India and Ease of Doing Business of Central Board of
Excise and Customs.

4. Within the context of international laws and treaties, an exemption that allows persons or entities to
continue with activities or operations that were approved before the implementation of new rules,
regulations or laws is known as;
(a) Grandfather clause
(b) Retrospective clause
(c) Limitation of benefits clause
(d) General purpose test

Answer (a)
Explanation
A Grandfather clause is a situation in which an old rule continues to apply to some existing situations,
while a new rule will apply to all future situations. Frequently, the exemption is limited; it may extend
for a set period of time, or it may be lost under certain circumstances. An exemption that allows
persons or entities to continue with an activity they were engaging in before but the same activity is
not allowed to new entities.

The Double Taxation Avoidance Agreement Amendment protocol signed on May 10, 2016, between
India and Mauritius amends the existing treaty to provide India the right to tax the capital gains
arising to a Mauritian resident from alienation of shares of a company resident in India, if such shares
are acquired after April 1, 2017. Shares acquired up to March 31, 2017, are ‘grandfathered’ such that
its sale anytime in the future would be tax-protected as under the original India-Mauritius tax treaty.
The grandfathering provisions are in line with the spirit of refraining from retrospective legislation. It
can be anticipated that this move would potentially surge investments in India until March 31, 2017 to
take advantage of the grandfathering window.

5. The Most Favoured Nation (MFN) principle of World Trade Organisation (WTO) was in news recently.
Which of the following statements is/are correct in this context?
1. According to the MFN principle, each of the WTO member countries should “treat all the other
members equally as ‘most-favoured’ trading partners.”
2. MFN status entails special treatment for some nations, where the granting nation has a trade
advantage.
3. India accorded MFN status to Pakistan in 1996 as per India’s commitments as a WTO member.
Select the correct answer using the code given below.
(a) 1 and 2 only (b) 1 and 3 only (c) 2 only (d) 1, 2 and 3
Answer (b)
Explanation
According to the MFN principle of the WTO’s General Agreement on Tariffs and Trade (GATT) — to
which India is a signatory/contracting party — each of the WTO member countries should “treat all
the other members equally as ‘most-favoured’ trading partners.” The WTO says, "Grant someone a
special favour (such as a lower customs duty rate) and you have to do the same for all other WTO
members." Hence, though MFN sounds like special treatment, in effect it is not special treatment and
it means non-discrimination. The MFN status was accorded to Pakistan in 1996 as per India’s
commitments as a WTO member. But Pakistan has not reciprocated, reportedly citing “non-tariff
barriers” erected by India as well as huge trade imbalance.

6. Which of the following is/are included in the twelve indicators used by the World Economic Forum to
compute the Global Competitiveness Report?
1. Infrastructure.
2. Ease of doing business.


3. Technological readiness.
Select the correct answer using the code given below.
(a) 2 only (b) 2 and 3 only (c) 1, 2 and 3 (d) 1 and 3 only
Answer (d)
Explanation
The Global Competitiveness Report is a yearly report published by the World Economic Forum since
2004.The report assesses the competitiveness landscape of world economies, providing insight into the
drivers of their productivity and prosperity. The report analyses competitiveness along 12 pillars:
institutions, infrastructure, macroeconomic environment, health and primary education, higher
education and training, goods market efficiency, labour market efficiency, financial market
development, technological readiness, market size, business sophistication and innovation.
These are, in turn, organized into three sub-indices in line with three main stages of development:
basic requirements, efficiency enhancers, and innovation and sophistication factors.

7. With reference to the Revised Model Text for the Indian Bilateral Investment Treaty approved by the
Union Cabinet, which of the following statements is/are not correct?
1. It aims to maintain a balance between the investor’s rights and the Government obligations.
2. The refined Investor State Dispute Settlement (ISDS) provision in the model treaty completely bans
investors from commencing international arbitration.
3. The treaty has included taxation, compulsory licenses and national security in its purview.
Select the correct answer using the code given below.
(a) 1 only (b) 2 only (c) 3 only (d) 2 and 3 only
Answer (d)
Explanation
The Union Cabinet approved the revised Model Text for the Indian Bilateral Investment Treaty in
December 2015. The revised Indian model text for Bilateral Investment Treaty (BIT) will replace the
existing Indian Model BIT and will be used for re-negotiation of existing BITs and negotiation of future
BITs and investment chapters in Comprehensive Economic Cooperation Agreements (CECAs)/
Comprehensive Economic Partnership Agreements (CEPAs) / Free Trade Agreements (FTAs). The new
Indian Model BIT text will provide appropriate protection to foreign investors in India and Indian
investors in the foreign country, in the light of relevant international precedents and practices, while
maintaining a balance between the investor’s rights and the Government obligations.
The essential features of the model BIT include an “enterprise” based definition of investment, non-
discriminatory treatment through due process, national treatment, protections against
expropriation, a refined Investor State Dispute Settlement (ISDS) provision requiring investors to
exhaust local remedies before commencing international arbitration, and limiting the power of the
tribunal to awarding monetary compensation alone. The model excludes matters such as
government procurement, taxation, subsidies, compulsory licenses and national security to
preserve the regulatory authority for the Government.

8. The Public Sector Undertakings (PSUs) for strategic sale is identified by


(a) Department of Public Enterprises
(b) Department of Investment and Public Asset Management
(c) NITI Aayog
(d) Respective administrative Ministries

Answer (c)
Explanation
Strategic sale of Public Sector Undertakings (PSUs) refers to strategic disinvestment. Strategic
disinvestment means sale of substantial portion of Government shareholding in identified CPSEs or
PSUs upto 50 per cent or more, along with transfer of management control. Presently, NITI Aayog is
responsible for identifying PSUs for strategic sales and monitoring the closure of loss making
firms, a job being implemented by the Department of Investment and Public Asset Management
(DIPAM). Besides NITI Aayog is also giving approvals to the state-run entities to set up financial
joint ventures, a power which rested with company boards. The Department of Public
Enterprises(DPE), which continues to be the nodal authority for all PSUs, has recently come out
with guidelines asking the PSUs to seek ‘concurrence’ from NITI Aayog for such joint ventures.

9. With reference to the ‘Strategic Stake Sale’ of Government of India for Financial Year 2017, which of
the following statements is/are correct?
1. It involves the disposal of controlling stakes in listed Public sector Companies only.
2. In listed Public sector Companies, the government will lower its holding to less than 49% to free
them from state control.


Select the correct answer using the code given below.
(a) 1 only (b) 2 only (c) Both 1 and 2 (d) Neither 1 nor 2
Answer (b)
Explanation
According to the Niti Aayog report on strategic disinvestment in public sector units (PSUs), it involves
the stake sale of both listed and unlisted companies. Hence India is preparing a plan for big-ticket
asset sales that involves the disposal of controlling stakes in 22 listed and unlisted companies as the
Centre looks to meet the full-year disinvestment target of Rs 56,500 crore. In the case of unlisted
ones, the government will exit completely, offering 100% equity. In the case of listed ones, the
government will lower its holding to less than 49% to free them from state control.
On the list are large state-run companies such as Container Corporation of India, Bharat
Earthmovers, as many as three plants of the Steel Authority of India and unlisted entities like Cement
Corporation of India. On the list are companies that are profitable and some that aren’t but have big
asset bases. The key criterion is that none of them are engaged in areas that are strategically
critical for India.

10. Recently the Government of India introduced labour reforms in the textile sector in order to beat
competition from countries like Bangladesh and Vietnam. Which one of the following is not a feature of
these reforms?
(a) Overtime not to exceed eight hours per week
(b) Introduction of fixed term employment
(c) Centre to bear employee’s contribution of 12% to EPF for three years
(d) None of the above
Answer (c)
Explanation
The Centre announced a `6,000-crore special package, with tax and production incentives, for the
textile and apparel sector to enable domestic firms to compete globally. The package aims to help in
creating one crore jobs, mostly for women, in the next three years. A key element of the policy is
overtime of up to eight hours a week, translating into nearly 90 hours over three months compared to
the current norm of 50 hours over this period. The centre also introduced fixed term employment
which was an industry demand to deal with the seasonal nature of demand. A fixed term workman will
be considered on a par with a permanent workman in terms of working hours, wages, allowances and
other statutory dues. The government would bear the entire 12% of the employer's contribution of
the EPF scheme (not employees contribution) for new garment industry employees who are earning
less than `15,000 per month, for their first three years of employment.

11. With reference to Factories Act of 1948, consider the following statements:
1. It is applicable to establishments with ten or more workers, if the premise is using power.
2. It is applicable to establishments with twenty or more workers, without electricity connection.
Which of the statements given above is/are correct?
(a) 1 only (b) 2 only (c) Both 1 and 2 (d) Neither 1 nor 2
Answer (c)
Explanation
The Factories Act is a legislation that deals with safety, health and welfare of workers. The
present Factories Act is applicable to establishments with 10 or more workers, if the premise is
using power and to establishments with 20 or more workers, without electricity connection.
However, all factories that manufactures or deals with “hazardous substance and processes and
dangerous operations will be covered under this Act even if they employ a single worker.”

12. With reference to ‘Viability Gap Funding’ sometimes seen in the news, which of the statement given
below is/are correct?
1. It means a grant to support projects that are economically unjustified but are commercially viable.
2. In India viability gap funding is generally done by the Government to support the completion of
infrastructure projects.
Select the correct answer using the code given below.
(a) 1 only (b) 2 only (c) Both 1 and 2 (d) Neither 1 nor 2
Answer (b)
Explanation
‘Viability Gap Funding’ means a grant to support projects that are economically justified but not
commercially viable. The main constraint in India’s infrastructure sector is the lack of source for
finance. Some projects may not be financially viable though they are economically justified and
necessary. This is the nature of several infrastructural projects which are long term and development


oriented. For the successful completion of such projects, the government has designed Viability Gap
Funding (VGF). The scheme is designed as a Plan Scheme to be administered by the Ministry of
Finance and amount in the budget are made on a year-to- year basis. Such a grant under VGF is
provided as a capital subsidy to attract the private sector players to participate in PPP projects that are
otherwise financially unviable. Projects may not be commercially viable because of long gestation
period and small revenue flows in future. The VGF scheme was launched in 2004 to support projects
that come under Public Private Partnerships.

13. The ‘RuPay debit card’ is sometimes seen in the news. Which of the following entity developed the
card?
(a) Reserve Bank of India
(b) National Payments Corporation of India (NPCI)
(c) State Bank of India
(d) Clearing Corporation of India Ltd. (CCIL)
Answer (b)
Explanation
RuPay is India’s own card payment scheme. Like all other card payment schemes, notably Visa and
MasterCard, it was created to ensure cashless transactions. The scheme was developed by National
Payments Corporation of India (NPCI) an initiative of the Reserve Bank of India and an umbrella
institution for all retail payment systems in the country.

14. Which of the following is/are included in the ten doing business topics used by the World Bank to
compute the Ease of Doing Business Ranking?
1. Getting electricity
2. Getting credit
3. Paying taxes
4. Enforcing contracts
5. Trading across borders
Select the correct answer using the code given below.
(a) 1 and 2 only (b) 3 and 5 only (c) 1, 2, 3 and 5 only (d) 1, 2, 3, 4 and 5
Answer (d)
Explanation
The Doing Business report published by the World Bank presents results for two aggregate measures:
the distance to frontier score and the ease of doing business ranking, which is based on the distance
to frontier score. Economies are ranked on their ease of doing business, from 1–190. A high ease of
doing business ranking means the regulatory environment is more conducive to the starting and
operation of a local firm. The rankings are determined by sorting the aggregate distance to frontier
scores on 10 topics, each consisting of several indicators, giving equal weight to each topic. The ten
topics are
1. Starting a business
2. Dealing with construction permits
3. Getting electricity
4. Registering property
5. Getting credit
6. Protecting minority investors
7. Paying taxes
8. Trading across borders
9. Enforcing contracts
10. Resolving insolvency

15. Both Employees Provident Fund (EPF) and National Pension System (NPS) are related to retirement
benefits. Which one of the following statements best represents an important difference between the
two?
(a) Government bears the burden of paying a fixed rate of return every year for EPF, while NPS is
market-linked and returns are based on the performance of the funds
(b) EPF is tax-free, while for NPS tax is imposed on withdrawal
(c) EPF is provided only for the unorganized sector workers, while NPS is provided only for the
organized sector workers
(d) Both (a) and (b)
Answer (d)
Explanation
The Employees' Provident Fund provides for the institution of provident funds for employees in
factories and other establishments. The Act and Schemes framed under EPF are administered by a tri-


partite Board known as the Central Board of Trustees, Employees' Provident Fund, The Board
administers a contributory provident fund, pension scheme and an insurance scheme for the
workforce engaged in the organized sector in India. EPF accounts are statutory for firms hiring 20
employees or more, and are funded by employees paying 12 percent of their salary to the EPFO and
employers putting in a similar amount.
The National Pension System (NPS) was launched on 1st January, 2004 with the objective of providing
retirement income to all the citizens. NPS aims to institute pension reforms and to inculcate the habit
of saving for retirement amongst the citizens. Initially, NPS was introduced for the new government
recruits (except armed forces). With effect from 1st May, 2009, NPS has been provided for all citizens of
the country including the unorganised sector workers on voluntary basis. Unlike the EPF in which
the government bears the burden of paying a fixed rate of return every year, the NPS is market-
linked and returns are based on the performance of the funds. For the EPF, the entire
withdrawal is tax-free, while for NPS certain percentage of the withdrawal would be taxed.

16. With reference to the demonetization of currency in India, consider the following statements:
1. Demonetization is the act of stripping a currency unit of its status as legal tender.
2. In November 2016, the Government of India went for demonetisation of currency for the first time
after independence, by withdrawing ₹1000 and ₹500 notes.
Which of the statements given above is/are correct?
(a) 1 only (b) 2 only (c) Both 1 and 2 (d) Neither 1 nor 2
Answer (a)
Explanation
Demonetization is the act of stripping a currency unit of its status as legal tender (coins or banknotes
that must be accepted if offered in payment of a debt). On November 8, Prime Minister of India
announced that ₹500 and ₹1000 notes would cease to be legal tender at the stroke of midnight. But
this is not the first time that this step has being taken. In January 1946, the government
withdrew ₹1,000 and ₹10,000 bank notes. However, in 1954, the higher denomination notes of ₹1,000,
₹5,000 and ₹10,000 were reintroduced. Then, in January 1978, higher denomination notes of ₹1,000,
₹5,000 and ₹10,000 were again withdrawn during Janata Party rule to check black money circulation.
Now, in 2016, the NDA government has once again taken such a step
• to curb black money,
• to lower cash circulation in the country which is directly related with corruption,
• to eliminate fake currency and dodgy funds which have been used to finance terrorism

17. With reference to ‘National Food Security Act (NFSA)’, which of the following statements is/are correct?
1. The eligible persons will be entitled to receive 5 Kgs of foodgrains per person per month at
subsidised prices.
2. One clear advantage of implementing the NFSA is that the computerization and Aadhaar seeding
process may result in the elimination of bogus cards and beneficiaries.
3. Kerala and Karnataka were the last two states to implement NFSA.
Select the correct answer using the code given below.
(a) 1 only (b) 2 and 3 only (c) 1 and 2 only (d) 1, 2 and 3
Answer (c)
Explanation
As passed by the Parliament, Government has notified the National Food Security Act, 2013 on 10th
September, 2013 with the objective to provide for food and nutritional security in human life cycle
approach, by ensuring access to adequate quantity of quality food at affordable prices to people to
live a life with dignity. The Act provides for coverage of upto 75% of the rural population and upto
50% of the urban population for receiving subsidized foodgrains under Targeted Public Distribution
System (TPDS), thus covering about two-thirds of the population. The eligible persons will be entitled
to receive 5 Kgs of foodgrains per person per month at subsidised prices of Rs. 3/2/1 per Kg for
rice/wheat/coarse grains. The existing Antyodaya Anna Yojana (AAY) households, which constitute
the poorest of the poor, will continue to receive 35 Kgs of foodgrains per household per month.
Over the last year or so, Kerala and Tamil Nadu had been warming to the idea of joining the NFSA,
but sought time to do so. They wanted to complete end-to-end computerisation of their PDS and the
process of seeding Aadhaar numbers with family cards. One clear advantage of joining the national
food security system now is that the computerisation and Aadhaar seeding process may result in the
elimination of bogus cards and beneficiaries. By threatening to raise the price at which it was
allocating foodgrains if they did not implement the law, the Centre has managed to get Tamil Nadu and
Kerala to agree to the implementation of the Act from November 1.


18. The RBI has asked banks to set aside 100 percent of the deposits accrued between September 16 and
November 11 as incremental Cash Reserve Ratio (CRR) in order to address excess liquidity. Which of
the following statements is/are correct in this context?
1. Cash Reserve Ratio (CRR) is the average daily balance that a bank shall maintain with the Reserve
Bank as a share of such per cent of its Net Demand and Time Liabilities that the Reserve Bank
may notify from time to time.
2. Currently Banks earn 4% interest on CRR balances kept with the RBI.
3. Cash Reserve Ratio (CRR) can be held as cash, gold or as bonds.
Select the correct answer using the code given below.
(a) 1 and 2 only (b) 1 and 3 only (c) 1 only (d) 1, 2 and 3
Answer (c)
Explanation
Cash Reserve Ratio (CRR) is the average daily balance that a bank shall maintain with the Reserve
Bank as a share of such per cent of its Net Demand and Time Liabilities that the Reserve Bank may
notify from time to time. It is a specified minimum fraction of the total deposits of customers, which
commercial banks have to hold as reserves either in cash or as deposits with the central bank.
Banks do not earn any interest on CRR balances kept with the RBI.

19. With reference to ‘Food Safety and Standards Authority of India (FSSAI)’; consider the following
statements:
1. It lays down mechanisms and guidelines for accreditation of certification bodies engaged in
certification of food safety management system for food businesses.
2. Ministry of Food Processing Industries is the Administrative Ministry for the implementation of
FSSAI.
3. It provides scientific advice and technical support to Central Government and State Governments
in the matters of framing the policy and rules in areas which have a direct or indirect bearing of
food safety and nutrition.
Which of the statements given above is/are correct?
(a) 1 only (b) 2 and 3 only (c) 1 and 3 only (d) 1, 2 and 3
Answer (c)
Explanation
The Food Safety and Standards Authority of India (FSSAI) has been established under Food Safety
and Standard Act, 2006. FSSAI has been created for laying down science based standards for articles
of food and to regulate their manufacture, storage, distribution, sale and import to ensure availability
of safe and wholesome food for human consumption. Ministry of Health & Family Welfare,
Government of India is the Administrative Ministry for the implementation of FSSAI.
FSSAI has been mandated by the FSS Act, 2006 for performing the following functions:
• Framing of Regulations to lay down the Standards and guidelines in relation to articles of food and
specifying appropriate system of enforcing various standards thus notified.
• Laying down mechanisms and guidelines for accreditation of certification bodies engaged in
certification of food safety management system for food businesses.
• Laying down procedure and guidelines for accreditation of laboratories and notification of the
accredited laboratories.
• To provide scientific advice and technical support to Central Government and State Governments
in the matters of framing the policy and rules in areas which have a direct or indirect bearing of
food safety and nutrition.
• Collect and collate data regarding food consumption, incidence and prevalence of biological risk,
contaminants in food, residues of various, contaminants in foods products, identification of
emerging risks and introduction of rapid alert system.
• Creating an information network across the country so that the public, consumers, Panchayats etc
receive rapid, reliable and objective information about food safety and issues of concern.
• Provide training programmes for persons who are involved or intend to get involved in food
businesses.
• Contribute to the development of international technical standards for food, sanitary and phyto-
sanitary standards.
• Promote general awareness about food safety and food standards.

20. The basic aim of the ‘Market Stabilization Scheme’ (MSS) bonds is
(a) to absorb the excess liquidity in the banking system
(b) to raise extra-long term funds for the infrastructure sector
(c) to inject sufficient liquidity in the banking system
(d) to raise funds from overseas for supporting Make in India initiative


Answer (a)
Explanation
Market Stabilisation Scheme (MSS) bonds are special bonds floated on behalf of the government by
the RBI for the specific purpose of mopping up the excess liquidity in the system when regular
government bonds prove inadequate. These are mostly shorter-tenure bonds, of less than six
months’ maturity. The sudden surge in deposits due to the surrender of demonetised currency
notes in large quantities skews bond yields and interest rates, disrupting the functioning of the
market. To impound the excess liquidity, bankers felt MSS bonds were a better option than a hike in
CRR holdings. CRR doesn’t fetch any return for the depositing bank but the MSS bonds earn a
return and qualify for statutory liquidity ratio (SLR). The investing of excess money in the MSS
bonds also prevents the banks from cutting their deposit rates sharply to protect their earnings.

21. With reference to ‘Ujwal DISCOM Assurance Yojna’ (UDAY) scheme unveiled by Government of India,
consider the following statements:
1. Under the scheme, States shall take over the 50% of DISCOMs (Power Distribution Companies)
debt as on 30 September 2015 over two years.
2. The DISCOM debt taken over by the States will not include in the calculation of the fiscal deficit in
2015-16 and 2016-17.
3. UDAY is mandatory for all the States.
Which of the statements given above is/are correct?
(a) 1 and 3 only (b) 2 only (c) 2 and 3 only (d) 1, 2 and 3
Answer (b)
Explanation
UDAY is a Central scheme for the financial revival of State electricity distribution utilities. The
Memorandum of Understanding (MoU) of UDAY is a tri-partite agreement between Union Ministry of
Power, State Government and their respective discom(s). States shall take over 75% of DISCOM debt
as on 30 September 2015 over two years - 50% of DISCOM debt shall be taken over in 2015-16 and
25% in 2016-17. Government of India will not include the debt taken over by the States as per the
above scheme in the calculation of fiscal deficit of respective States in the financial years 2015-16 and
2016-17. UDAY is optional for all States. However, States are encouraged to take the benefit at the
earliest as benefits are dependent on the performance.

22. The term ‘Marginal Standing Facility’ is sometimes seen in the news. Which of the following statements
best describes/describe this term?
1. It is a detailed procedure by which a bank with huge non-performing assets is taken over by
another bank.
2. It helps the commercial banks to borrow from the RBI in certain emergencies when the inter-bank
liquidity dries up.
3. It is to contain the volatility in the overnight interbank rates.
Select the correct answer using the codes given below.
(a) 1 only (b) 2 and 3 only (c) 1 and 3 only (d) 1, 2 and 3
Answer (b)
Explanation
Marginal Standing Facility (MSF) rate is a new liquidity adjustment facility window created by RBI in
its credit policy of May 2011. MSF is the rate the banks are able to borrow over-night funds from RBI
against its approved government securities. This window was created for the commercial banks to
borrow from the RBI in certain emergencies when the inter-bank liquidity dries up and there is
volatility in the overnight interest rates. To curb this volatility, RBI allowed them to pledge G-secs and
get more funds from RBI at a rate higher than the repo rate. The overall idea behind the MSF is to
contain the volatility in the overnight interbank rates. The rate of interest of MSF is set 50 bps above
the repo rate. The banks can borrow up to 1% of their net demand and time liabilities from this facility.

23. Which of the following is/are the feature/features of Exchange Traded Funds (ETF)?
1. Low expense ratio
2. More Transparency
3. Passive investment
Select the correct answer using the code given below.
(a) 1 only (b) 2 and 3 only (c) 1 and 3 only (d) 1, 2 and 3
Answer (d)
Explanation
Exchange Traded Funds (ETFs), as their name denotes, are mutual funds that are traded on stock
exchanges and bought and sold like stocks. They mostly track an underlying index (hence more


transparent) and are passive investment tools that is, the fund manager has no role to play in ETFs.
The expense ratio is therefore, much lower in ETFs.

24. Regarding ‘Purchasing Managers Index’ (PMI) often seen in news, which of the following statements
is/are correct?
1. PMI is calculated for service sector only.
2. A reading above 50 indicates economic expansion and below 50 indicates contraction of the
economy.
Select the correct answer using the code given below.
(a) 1 only (b) 2 only (c) Both 1 and 2 (d) Neither 1 nor 2
Answer (b)
Explanation
The Purchasing Managers' Index (PMI) is an indicator of the economic health of the manufacturing
sector as well as service sector. Its survey is usually carried about among the private sector
companies. PMI reading above 50 indicates economic expansion and below 50 reading gives
contraction nature of the economy. Recently due to the demonetisation effect, Nikkie India Service
PMI has contracted below 50-point mark showing the economic slowdown.

25. With reference to the ‘Cash Management Bills’, consider the following statements:
1. These are long term capital market instrument.
2. These are issued by Government of India in consultation with RBI.
3. Government of India recently issued Cash Management Bills worth ₹60,000 crore under
governments Market Stabilisation
4. Scheme (MSS) to mop up extra liquidity from the system against the backdrop of demonetisation.
Which of the following statements given above is/are correct?
(a) 1 only (b) 2 and 3 only (c) 1 and 3 only (d) 1, 2 and 3
Answer (b)
Explanation
The Government of India, in consultation with the Reserve Bank of India, issues a short-term
instrument, known as Cash Management Bills, to meet the temporary cash flow mismatches of the
Government. The Cash Management Bills will be non-standard, discounted instruments issued for
maturities less than 91 days.

26. Which one of the following is not the service provided by ‘Goods and Service Tax Network’ (GSTN)?
(a) It establishes a uniform interface for the tax payer and a common and shared IT infrastructure
between the Centre and States.
(b) It integrates the entire direct tax ecosystem so as to bring all the tax administrations (Centre, State
and Union Territories) to the same level of IT maturity with uniform formats and interfaces for
taxpayers and other external stakeholders.
(c) It carries out research; study best practises and provide training and consultancy to the tax
authorities and other stakeholders.
(d) It will assist tax authorities in improving tax compliance and transparency of tax administration
system.
Answer (b)
Explanation
Goods and Services Tax Network (GSTN) is a Section 8 (under new companies Act, not for profit
companies are governed under section 8), non-Government, private limited company. The Government
of India holds 24.5% equity in GSTN and all States of the Indian Union, including NCT of Delhi and
Puducherry, and the Empowered Committee of State Finance Ministers (EC), together hold another
24.5%. Balance 51% equity is with non-Government financial institutions. It would involve integrating
the entire indirect tax ecosystem so as to bring all the tax administrations (Centre, State and Union
Territories) to the same level of IT maturity with uniform formats and interfaces for taxpayers and
other external stakeholders.
The GSTN has been mandated to provide the following additional services to the various stakeholders:
• Provide common and shared IT infrastructure and services to the Central and State
Governments, tax payers and other stakeholders.
• Partner with other agencies for creating an efficient and user-friendly GST Eco-system.
• Encourage and collaborate with GST Suvidha Providers (GSPs) to roll out GST applications for
providing simplified services to the stakeholders.
• Carry out research, study best practises and provide training and consultancy to the Tax
authorities and other stakeholders.


• Provide efficient backend services to the tax departments of the Central and State Governments
on request.
• Develop Tax Payer Profiling Utility (TPU) for Central and State Tax Administration.
• Assist tax authorities in improving tax compliance and transparency of tax administration
system.
• Deliver any other services of relevance to the Central and State Governments and other
stakeholders on request.

27. Which of the following statements are true in relation to the revised India – Cyprus Double Taxation
Avoidance Agreement (DTAA)?
1. New DTAA provides for source based taxation of capital gains arising from alienation of shares,
instead of residence based taxation provided under the existing DTAA.
2. A grandfathering clause has been provided for investments made prior to 1st April, 2017, in respect
of which capital gains would continue to be taxed in the country of which taxpayer is a resident.
3. The amendments provide that the shares acquired between April 1, 2017 and March 31, 2019 will
attract capital gains tax at a 50% discount on the domestic tax rate.
4. The new pact expands the scope of ‘permanent establishment’ and reduces the tax rate on royalty
in the country from which payments are made to 10% from the existing rate of 15%, in line with
the tax rate under Indian tax laws.
Select the correct answer using the codes given below.
(a) 1, 2 and 3 (b) 1, 2 and 4 (c) 1, 3 and 4 (d) 2, 3 and 4
Answer (b)
Explanation
A revised Agreement between India and Cyprus for the Avoidance of Double Taxation and the
Prevention of Fiscal evasion (DTAA) with respect to taxes on income, along with its Protocol, will
replace the existing DTAA that was signed by two countries on 13th June 1994. This step follows the
recent amendment of the Double Taxation Avoidance Agreement with Mauritius. As in the case of
Mauritius, the treaty with Cyprus had provided for residence-based taxation of capital gains. With the
revision of the treaty now approved by the Cabinet, capital gains will be taxed in India for entities
resident in Cyprus, subject to double tax relief. In other words, India will have the right to tax capital
gains arising in India. Though this will be effective prospectively, similar to the provision made in the
India-Mauritius treaty, it does not provide for a transitionary period like the one provided by the India-
Mauritius treaty, wherein only half the capital gains tax rate will be applicable between 2017 and
2019.

28. What does Angel Investor mean to economy?


(a) Investor providing short-term capital to industries
(b) Investor providing long-term start-up capital to new entrepreneurs
(c) SEBI registered investors in the Capital Bond Market
(d) Investor who provide fund to sick industries
Answer (b)
Explanation
Angel Investors are a class of well-to-do investors, usually experienced industry folk who take equity
stakes in start-ups. Recently Siliguri-based start-up Teabox, which caters to a worldwide market of
tea-lovers, has secured an undisclosed amount of funding from industrialist and prolific angel investor
Ratan Tata. In India, Angel funds are regulated by SEBI.

29. Consider the following statements regarding the ‘Cash Reserve Ratio (CRR)’:
1. The CRR for Scheduled Commercial Banks have no ceiling rate.
2. The banks get a minimum interest for the money they parked under CRR with Reserve Bank of
India.
Which of the statements given above is/are correct?
(a) 1 only (b) 2 only (c) Both 1 and 2 (d) Neither 1 nor 2
Answer (a)
Explanation
Cash Reserve Ratio (CRR) is a specified minimum fraction of the total deposits of customers, which
commercial banks have to hold as reserves either in cash or as deposits with the central bank. CRR is
set according to the guidelines of the central bank of a country. CRR is a crucial monetary policy tool
and is used for controlling money supply in an economy. The banks get no interest for the money they
parked with RBI under CRR.
Earlier RBI was empowered to fix CRR between 3-20% by notification. However, from 2006 onwards
the RBI is empowered to fix the CRR on its discretion without any ceiling.


30. With reference to ‘Goods and Service Tax Council (GSTC)’, consider the following statements:
1. GSTC is an extra constitutional body.
2. It is headed by Union Finance Minister.
3. Vice Chairman of NITI Aayog is its ex-officio member.
Which of the statements given above is/are correct?
(a) 1 only (b) 2 and 3 only (c) 2 only (d) 1, 2 and 3
Answer (c)
Explanation
As per Constitution (One Hundred and First Amendment) Act, 2016, the President of India has
constituted the Goods and Services Tax Council. The Goods and Services Tax Council shall consist of
the following members’ viz. Union Finance Minister as the Chairperson, Union Minister of State in
charge of Revenue as member and the Minister in charge of finance or revenue or any other minister
nominated by each State Government as other members.
The decisions of the GST Council are made by three-fourth majority of the votes cast. The Centre has
one-third of the votes cast, and the States together have two-third of the votes cast. Each State has
one vote, irrespective of its size or population.
Apart from the above:
• The Secretary (Revenue) will be appointed as the Ex-Officio Secretary to the GST Council.
• The Chairperson, Central Board of Excise and Customs (CBEC), will be included as a
permanent invitee (non-voting) to all proceedings of the GST Council.
• One post of Additional Secretary to the GST Council in the GST Council Secretariat (at the level
of Additional Secretary to the Government of India) will be created.
• Four posts of Commissioner in the GST Council Secretariat (at the level of Joint Secretary to
the Government of India) will also be created.

31. How does the ‘Insolvency and Bankruptcy Code, 2016’ benefit the business environment in the
country?
1. By creating time-bound processes for insolvency resolution of companies and individuals.
2. By furthering credit expansion.
3. By improving the recovery rate of Asset Reconstruction Companies (ARCs).
Select the correct answer using the codes given below.
(a) 1 and 2 only (b) 2 only (c) 1 and 3 only (d) 1, 2 and 3
Answer (d)
Explanation
Insolvency and Bankruptcy Code, 2016 (IBC 2016) aims to consolidate and amend the laws relating
to reorganisation and insolvency resolution of corporate persons, partnership firms and
individuals in a time bound manner for maximisation of value of assets of such persons, to promote
entrepreneurship, availability of credit and balance the interests of all the stakeholders including
alteration in the order of priority of payment of Government dues and to establish an Insolvency
and Bankruptcy Board of India (IBBI).
The resolution processes will be conducted by licensed insolvency professionals (IPs). These IPs will
be members of insolvency professional agencies (IPAs). Information utilities (IUs) will be
established to collect, collate and disseminate financial information to facilitate insolvency resolution.
Proper implementation of IBC 2016 can free the capital locked up as bad loans, which could be
utilised for further credit expansion. Time bound resolution would improve the recovery rate of
Asset Reconstruction Companies and combined with other structural reforms it is expected to
improve India’s Ease of Doing Business and Global Competiveness Ranking significantly.

32. Which one of the following is not a feature of Google tax?


(a) It is the tax levied on the advertising revenues of multinational web companies.
(b) It is a tax on both Business to Business (B2B) and Business to Customer (B2C) services.
(c) India became the first nation to tax digital transactions after introduction of Google tax.
(d) It is basically an idea from Action plan 1 of OECD’s Base Erosion and Profit Shifting (BEPS).
Answer (b)
Explanation
India became the first nation to tax digital transactions when it introduced the equalisation levy of 6%
— also known as the Google tax — on the advertising revenue of multinational web companies in June
2016. According to the Budget announcement, any person or entity that makes a payment exceeding
`1 lakh in a financial year to a non-resident technology company will now need to withhold 6% tax on
the gross amount being paid as an equalisation levy. This tax, however, is only applicable when the
payment has been made to avail certain Business to Business (B2B) services that cover online


advertisements, provision for digital advertising space or any other facility or service for the
purpose of online advertisements. This list will expand soon. The idea of an equalization levy comes
from Action 1 of the Organisation for Economic Cooperation and Development’s (OECD’s) base erosion
and profit shifting (BEPS) Action Plan. The Action plan considers equalisation levy as an option to tax
digital transactions.

33. With reference to bond market, consider the following statements:


1. As prices of bonds rise, their yields increase.
2. As prices of bonds rise, their yields decrease.
Which of the statements given above is/are correct?
(a) 1 only (b) 2 only (c) Both 1 and 2 (d) Neither 1 nor 2
Answer (b)
Explanation
Yield is a figure that shows the return you get on a bond. If you buy a bond worth ₹100 with a 10%
interest rate, after 1 year you will get a bond yield of ₹10. But if the bond price goes down to ₹90 for
some reasons, then the bond yield is increased to 11.11%. This happens because you are getting the
same guaranteed ₹10 on an asset that is worth ₹90. It shows that the bond yield and the bond prices
is having an inverse relationship.

34. The term Gross Value Added at basic prices represents


(a) Gross Domestic Product at market prices minus Product taxes plus Product subsidies
(b) Gross Domestic Product at market prices plus Product taxes minus Product subsidies
(c) Gross Domestic Product at market prices minus Production taxes plus Production subsidies
(d) Gross Domestic Product at market prices plus Production taxes minus Production subsidies
Answer (a)
Explanation
In the revision of National Accounts statistics done by Central Statistical Organization (CSO)
in January 2015, it was decided that sector-wise wise estimates of Gross Value Added (GVA) will now
be given at basic prices instead of factor cost. In simple terms, for any commodity the basic price
is the amount receivable by the producer from the purchaser for a unit of a product minus any tax
on the product plus any subsidy on the product. However, GVA at basic prices will include
production taxes and exclude production subsidies available on the commodity. On the other
hand, GVA at factor cost includes no taxes and excludes no subsidies and GDP at market prices
include both production and product taxes and excludes both production and product subsidies.
The relationship between GVA at Factor Cost and GVA at Basic Prices and GDP at market prices
and GVA at basic prices is shown below:
GVA at basic prices = GVA at factor cost + (Production taxes less Production subsidies)
GDP at market prices = GVA at basic prices + Product taxes - Product subsidies

35. With reference to ‘Capital Gains Tax’, which of the following statements is/are correct?
1. Tax rate on Short term capital gains in equity investment is zero per cent.
2. Equity investments held over one year attract Long Term Capital Gains Tax of 15 per cent.
Select the correct answer using the codes given below.
(a) 1 only (b) 2 only (c) Both 1 and 2 (d) Neither 1 nor 2
Answer (d)
Explanation
Gain arising on transfer of capital asset is charged to tax under the head “Capital Gains”. Income from
capital gains is classified as “Short Term Capital Gains” and “Long Term Capital Gains”. Any capital
held by a tax payer for a period of not more than 36 months immediately preceding the date of its
transfer is treated as the short-term capital asset and more than 36 months as long term capital asset.
But in case of listed shares, units of equity oriented mutual funds, listed securities like debentures
and listed securities Units of UTI and Zero Coupon Bonds, the period of holding to be considered is 12
months instead of 36 months.
At present, equity investments held over 1 year do not attract any long-term capital gains tax, while
the tax rate on short term capital gains is at 15 per cent.

36. Which of the following pairs are correctly matched?


1. Maharaja bonds : India
2. Panda bonds : Japan
3. Pro bonds : China
Select the correct answers using the codes given below.
(a) 1 only (b) 2 and 3 only (c) 1, 2 and 3 (d) 1 and 3 only


Answer (a)
Explanation
'Maharaja' bond, a rupee-denominated onshore debt instrument issued by International Finance
Corporation (IFC), an arm of World Bank. The Maharaja bonds have been issued to support the
country's domestic capital markets.
A Panda bond is a Renminbi (RMB) denominated bond issued in Mainland China. The issuer of Panda
bond is always a foreign entity, can be either a financial or non-financial institution. The first two
Panda Bonds were issued in October 2005, by IFC and Asian Development Bank (ADB).
Japan-based Pro-bonds, which came into existence in 2008, target professional investors and are
aimed at developing Japan's bond market. Through this route, Japanese institutional investors, who
are facing negative interest rates, can earn higher income. Recently, ICICI Bank, India's largest private
lender, has raised 10 billion Japanese yen ($88 million) by selling a new series of Pro-bonds.

37. The branches of which of the following banks have been authorised by RBI as ‘Currency chests’?
1. Scheduled Commercial Banks
2. Private Sector Banks
3. Small Finance Banks
4. Co-operative Banks
Select the correct answers using the codes given below.
(a) 1, 2 and 3 only (b) 1, 2 and 4 only (c) 2, 3 and 4 only (d) 1, 3 and 4 only
Answer (b)
Explanation
Currency chests are branches of selected banks authorised by RBI to stock the rupee notes and coins.
The responsibility for managing the currency in circulation is vested in the RBI. The RBI has setup
4,075 currency chests across the country and the PSU banks account 95% of it. The Private sector
banks (160 units), Co-operative banks (3 units), Foreign banks (4 units) and Regional Rural Banks (5
units) plays smaller role in stocking currency on behalf of RBI.

38. Which one of the following best describes the main objective of the ‘General Anti-Avoidance Rules’
(GAAR)?
(a) Helping the government to unearth the black money in the economy
(b) Providing a single window to declare the tax evaded income
(c) Empowering revenue authorities in a country to deny the tax benefits of transactions which do not
have any commercial substance
(d) Providing the right to MNCs to avail the ‘Limitation of Benefits’ under the Double Taxation
Avoidance Agreement
Answer (c)
Explanation
‘General Anti-Avoidance Rules’ (GAAR) is a set of rules designed to give Indian authorities the right to
scrutinize any tax transactions which they believe are structured solely to avoid taxes. If GAAR starts
in India, then any transaction that carries a tax benefit could be questioned. The taxman may want to
know whether the transaction was done in the normal course of business or conducted simply with an
intention to avoid taxes.
Recently Government of India has clarified that the ‘Limitation of Benefit’ clause under the recently
amended Double Taxation Avoidance Agreements with various countries would not invite the scrutiny
under the GAAR.

39. Regarding the Electoral Bonds, which one of the following statements is not correct?
(a) It will be issued only by a notified bank for specified denominations
(b) Political funding can be done by buying these bonds through digital payments or cheque only.
(c) It can be received by a registered political party only.
(d) The identity of the donor can be clearly known, shows its merit.
Answer (d)
Explanation
Electoral bonds will be issued by a notified bank for specified denominations. If you are keen to donate
to a political party, you can buy these bonds by making payments digitally or through cheque. You are
then free to gift the bond to a registered political party. The bonds will likely be bearer bonds and the
identity of the donor will not be known to the receiver.

40. Which of the following pairs are not correctly matched?


1. BSE: Asia’s first stock exchange
2. India INX: First international stock exchange of India


3. MCX: Largest commodity exchange in India
Select the correct answer using the code given below.
(a) 1 and 2 only (b) 2 only (c) 1 and 3 only (d) None
Answer (d)
Explanation
The Multi Commodity Exchange of India Limited (MCX), India’s first listed exchange, is a state-of-
the-art, commodity derivatives exchange that facilitates online trading, and clearing and settlement
of commodity futures transactions. It is the largest commodity exchange in the country.
India International Exchange (India INX) is India’s first International Exchange set up at
International Financial Services Centre at GIFT City. It is set up by BSE (Bombay Stock Exchange
Ltd.)
Established in 1875, BSE (formerly known as Bombay Stock Exchange Ltd.), is Asia's first exchange
& with more than 5500 listed companies, it is the world's No. 1 exchange in terms of listed companies.
BSE provides an efficient and transparent market for trading in equity, debt instruments,
derivatives, mutual funds. It also has a platform for trading in equities of small-and-medium
enterprises (SME).

41. Consider the following statements:


1. An interest rate future is a contract between the buyer and seller agreeing to the future delivery of
any interest-bearing asset.
2. Interest rate futures are used for hedging purposes and speculation purposes.
Which of the statements given above is/are correct?
(a) 1 only (b) 2 only (c) Both 1 and 2 (d) Neither 1 nor 2
Answer (c)
Explanation
An interest rate future is a futures contract with an underlying instrument that pays interest. An
interest rate future is a contract between the buyer and seller agreeing to the future delivery of any
interest-bearing asset. The interest rate future allows the buyer and seller to lock in the price of the
interest-bearing asset for a future date.
Interest rate futures are used for hedging purposes and speculation purposes.

42. Market capitalisation generally means


(a) the total market value of a company's outstanding shares based on current market price
(b) the total market value of a company's outstanding shares based on a base year price
(c) the total market value of shares made available by a company through IPO
(d) market borrowing by the Government through the sale of Government securities
Answer (a)
Explanation
Market capitalization refers the total market value of a company's outstanding shares. Commonly
referred to as "market cap," it is calculated by multiplying a company's shares outstanding by the
current market price of one share. The investment community uses this figure to determine a
company's size, as opposed to using sales or total asset figures.

43. Which of the following are the modes of disinvestment used by the Union Government for the 2016-17
financial year?
1. Further Public Offering (FPO)
2. CPSE Exchange Traded Fund (ETF)
3. Institutional Placement Program (IPP)
4. Delisting
Select the correct answer using the code given below.
(a) 1, 2, 3 and 4 (b) 2 and 4 only (c) 1 and 3 only (d) 1, 2 and 3 only
Answer (d)
Explanation
Methods of disinvestment of CPSEs
1. Initial Public Offering (IPO) – offer of shares by an unlisted CPSE or the Government out of its
shareholding or a combination of both to the public for subscription for the first time.
2. Further Public Offering (FPO) – offer of shares by a listed CPSE or the Government out of its
shareholding or a combination of both to the public for subscription.
3. Offer for sale (OFS) of shares by Promoters through Stock Exchange mechanism – method allows
auction of shares on the platform provided by the Stock Exchange; extensively used by the
Government since 2012.


4. Strategic sale - sale of substantial portion of the Government shareholding of a central public
sector enterprise (CPSE) of up to 50%, or such higher percentage as the competent authority may
determine, along with transfer of management control.
5. Institutional Placement Program (IPP) – only Institutions can participate in the offering.
6. CPSE Exchange Traded Fund (ETF) –Disinvestment through ETF route allows simultaneous sale
of GoI's stake in various CPSEs across diverse sectors through single offering. It provides a
mechanism for the GoI to monetize its shareholding in those CPSEs which form part of the ETF
basket.
Delisting involves removal of listed securities of a company from a stock exchange where it is traded
on a permanent basis.

44. Which of the following is/are the power trading market places in India?
1. Power Exchange of India Ltd.
2. India Energy Exchange
3. Multi Commodity Exchange
4. Bombay Stock Exchange
Select the correct answer using the code given below.
(a) 1 only (b) 2, 3 and 4 only (c) 1 and 2 only (d) 1, 2 and 3 only
Answer (c)
Explanation
In India, there is only two power trading market platforms viz. Power Exchange of India Ltd. and
India Energy Exchange.

45. Consider the following statements:


1. Full convertibility is a reason for international acceptance of US Dollar.
2. Indian rupee is fully convertible in capital account of Balance of Payments.
3. Full convertibility of Indian rupee will attract more foreign capital into the country.
Select the correct answer using the code given below.
(a) 1 and 2 only (b) 1 and 3 only (c) 2 and 3 only (d) 1, 2 and 3
Answer (b)
Explanation
Indian rupee is fully convertible in current account of the Balance of Payment only.

46. Consider the following statements regarding the changes in the income tax proposed in Budget 2017-
18:
1. All the earlier existing tax slabs are provided with 5% tax deduction.
2. Surcharge is levied only on those who earn income more than ₹1 Crore annually.
Select the correct answer using the code given below.
(a) 1 only (b) 2 only (c) Both 1 and 2 (d) Neither 1 nor 2
Answer (d)
Explanation
The Budget 2017-18 provided a tax relief to people whose annual income fall in between ₹2.5 lakhs
and ₹5 lakhs by cutting their tax rate to 5% from existing 10%. For those who earn ₹50 lakhs to
₹1 Crore annually, the Budget imposes additional surcharge of 10% of income tax. Already there is a
15% surcharge on income tax for those who earn ₹1 Crore or more annually.

47. What is/are the purpose/purposes of Government’s ‘DigiGaon’ initiative?


1. to provide telemedicine through digital technology
2. to provide skills through digital technology
3. to provide education through digital technology
Select the correct answer using the code given below.
(a) 1 only (b) 2 and 3 only (c) 1 and 3 only (d) 1, 2 and 3
Answer (d)
Explanation
The 2017-18 Union Budget has proposed ‘DigiGaon’ initiative provide telemedicine, education and
skills through digital technology.

48. Regarding Foreign Investment Promotion Board (FIPB), which of the following statements is/are
correct?
1. FIPB offers a single window clearance for applications on Foreign Direct Investment (FDI) in India
that are under the automatic route.


2. FIPB is housed in the Department of Economic Affairs, Ministry of Finance.
3. FIPB can clear the proposals of FDI worth ₹5000 Crore.
Select the correct answer using the code given below.
(a) 1 only (b) 2 and 3 only (c) 1 and 3 only (d) 1, 2 and 3
Answer (b)
Explanation
The Foreign Investment Promotion Board (FIPB) offers a single window clearance for applications on
Foreign Direct Investment (FDI) in India that are under the approval route. FIPB, housed in the
Department of Economic Affairs, Ministry of Finance, is an inter-ministerial body, responsible for
processing of FDI proposals and making recommendations for Government approval. The Minister of
Finance who is in-charge of FIPB would consider the recommendations of FIPB on proposals with
total foreign equity inflow of and below ₹5000 Crore. The recommendations of FIPB on proposals
with total foreign equity inflow of more than ₹5000 Crore would be placed for consideration of
Cabinet Committee on Economic Affairs (CCEA).

49. Twin Balance Sheet (TBS) problem’ include/includes


(a) Non-Performing Assets (NPAs) of Public Sector Banks
(b) Indebtedness of corporate sector
(c) Fiscal deficit of Union government
(d) Both (a) and (b)
Answer (d)
Explanation
The Twin Balance Sheet (TBS) challenge of Indian Economy includes both the Non-Performing Assets
of Public Sector Banks and the indebtedness of corporate sector of the economy.

50. How does the proposed Public Sector Asset Rehabilitation Agency (PARA) seek to improve the Indian
Economy?
1. By identifying the sick industries and proposing divestment
2. By addressing problems like NPAs of banks and indebtedness of corporates
3. By resuming the stalled infrastructure projects
Select the correct answer using the code given below.
(a) 1 and 2 only (b) 2 only (c) 1 and 3 only (d) 1, 2 and 3
Answer (b)
Explanation
According to the Economic survey 2016-17, the country is grappling with a 'Twin Balance Sheet' (TBS)
problem of over-leveraged companies and bad-loan-encumbered banks, a legacy of the boom years
around the global financial crisis. So Survey stressed on the need to establish a centralised Public
Sector Asset Rehabilitation Agency (PARA) to deal the same. The agency could take charge of the
largest, most difficult cases, and make politically tough decisions to reduce debt, the survey said.

51. Consider the following industries:


1. Textiles
2. Cement
3. Refinery products
4. Automobile
Which of these are core industries?
(a) 1 and 4 only (b) 2 and 3 only (c) 1, 2 and 3 only (d) 2, 3 and 4 only
Answer (b)
Explanation
The eight core industries include Coal, Crude oil, Cement, Electricity, Fertilisers, Natural Gas,
Refinery products, Steel.

52. Which of the following statements are not the major reforms associated with the Budget 2017-18?
1. Presentation of Budget advanced to 1st February to enable the Ministries to operationalize all
activities from the commencement of the financial year.
2. Merger of Railways Budget with General Budget to bring Railways to the centre stage of
Government’s Fiscal Policy.
3. Removal of revenue and capital classification of expenditure to facilitate a holistic view of
allocations for sectors and ministries.
Select the correct answer using the code given below.
(a) 1 and 2 only (b) 3 only (c) 2 and 3 only (d) None


Answer (b)
Explanation
Budget 2017-18 contains 3 major reforms. First, presentation of Budget advanced to 1st February to
enable the Ministries to operationalize all activities from the commencement of the financial year.
Second, merger of Railways Budget with General Budget to bring Railways to the centre stage of
Government’s Fiscal Policy and Third, removal of plan and non-plan classification of expenditure to
facilitate a holistic view of allocations for sectors and ministries.

53. Government of India advanced the Budget 2017-18 by a month


(a) to ensure that the budget process completes before the new financial year begins.
(b) to enable the ministries and departments to operationalize all the schemes right from the
commencement of the next financial year.
(c) to fully utilise the available working season before the onset of the monsoon.
(d) All of the statements (a), (b) and (c) given above is correct in this context.
Answer (d)
Explanation
Government of India advanced Budget 2017-18 by a month to ensure that the budget process
completes before the new financial year begins. It also enables the ministries and departments to
operationalize all the schemes right from the beginning of the next financial year without wasting their
precious time. It also help to utilise the available working season before the onset of the south west
monsoon in India.

54. ‘Swatchh Dhan Abhiyan’ sometimes seen in the news, is


(a) an NGO that works to ensure cleansing of political funding in India.
(b) a Government of India initiated programme to disclose the unaccounted income.
(c) an initiative of Income Tax Department that involves e-verification of large cash deposits made
during 9th November to 30th December 2016.
(d) the Universal Basic Income scheme proposed in Budget 2017-18
Answer (c)
Explanation
Income Tax Department (ITD) has initiated Operation Clean Money, or Swatchh Dhan Abhiyan on
31st January 2017. Initial phase of the operation involves e-verification of large cash deposits made
during 9th November to 30th December 2016. Data analytics has been used for comparing
the demonetisation data with information in ITD databases. In the first batch, around 18 lakh persons
have been identified in whose case, cash transactions do not appear to be in line with the tax payer’s
profile.

55. Regarding the Central Public Sector Enterprises (CPSE) ETF, which one of the following statement is
correct?
(a) It tracks the BSE S&P 500 Index
(b) It aims to boost the disinvestment programme of Government of India
(c) It includes the shares of Maharatna companies only
(d) Maximum weightage is held by Engineers India Ltd.
Answer (b)
Explanation
Central Public Sector Enterprises (CPSE) ETF, which tracks the Nifty CPSE Index, is a
concentrated portfolio of 10 stocks, whose main aim is to help the Government of India (GOI) in
disinvesting its stake in a few CPSEs via the ETF route. The portfolio comprises of 10 Maharatna and
Navaratna CPSEs whose weightage in the index are given below.


56. Which of the following statements regarding the proposed amendments in Budget 2017-18 in relation
to Airport Authority of India Act is not correct?
(a) The amendments enable effective monetization of land assets under the Airport Authority of India.
(b) The money raised from these can be used for upgradation of airports.
(c) They enable airports to sell land to developers who want to build multiplexes, shopping malls and
convention halls only.
(d) Effective monetisation will enhance the non-aeronautical revenues and thereby will reduce the cost
of flight charges.
Answer (c)
Explanation
The proposed amendment to the Airports Authority of India Act in Budget 2017-18 to allow
monetisation of land is unlikely to specify for what purpose the denotified land can be used. The
money so raised would be used for airport up-gradation.
At present, the use to which land at AAI airports can be put is specified under Section 12 of the AAI
Act 1994. The Act states that the land can be used for constructing residential buildings for its
employees, establishing and maintaining hotels, restaurants and rest rooms at or near the airports or
establish warehouses and cargo complexes at the airports for the storage or processing of goods or
arrange for postal, money exchange, insurance and telephone facilities for the use of passengers and
other persons at the airports and civil enclaves among others. The government is keen that the
proposed amendment to the AAI Act creates an ‘omnibus category’ which will try and take care of
use of land in perpetuity.
Change in the act will allow AAI to lease out land for various other purposes like shopping malls and
convention halls and help them to increase their non-aeronautical revenues, which would help to
reduce aeronautical charges on airlines, which will benefit the airlines as well as passengers.

57. Consider the following statements regarding the ‘Offer For Sale’ (OFS) mechanism:
1. Only listed companies can issue shares via OFS.
2. It requires no physical application.
3. Unlike FPO, it requires processing fee.
Which of the statements given above is/are correct?
(a) 1 only (b) 2 and 3 only (c) 1 and 3 only (d) 1, 2 and 3

Answer (d)
Explanation
Offer for sale (OFS) is a simpler method of share sale through the exchange platform for listed
companies. The mechanism was first introduced by India's securities market regulator SEBI, in
2012, to make it easier for promoters of publicly-traded companies to cut their holdings and comply
with the minimum public shareholding norms by June 2013. The OFS mechanism is used only when
existing shares are put on the block. The mechanism is available to 200 top companies in terms of
market capitalisation. Unlike FPO, the OFS requires no physical application but the processing fee.

58. Which of the following is not a feature of hard currency?


1. Widely accepted around the world
2. Highly liquid
3. Part of IMF’s SDR basket
Select the correct answer using the code given below.
(a) 1 and 2 only (b) 2 and 3 only (c) 3 only (d) 1 and 3 only

Answer (c)
Explanation
Hard currency is a currency widely accepted around the globe as a form of payment for goods and
services. A hard currency is expected to remain relatively stable through a short period of time, and
to be highly liquid in the forex, or foreign exchange market. A hard currency generally comes from a
country with a strong economic and political situation.
It is not essential that all hard currencies are part of IMF’s SDR basket.

59. With reference to the ‘Aadhaar Pay’, consider the following statements:
1. It is a digital payment platform.
2. Biometrics and mobile phones are essential to carry out the transactions under it.
Which of the statements given above is/are correct?
(a) 1 only (b) 2 only (c) Both 1 and 2 (d) Neither 1 nor 2


Answer (a)
Explanation
The Government of India plans to launch the biometric-based ‘Aadhaar Pay’ payment module
seeking to accelerate the drive toward digital-money transactions in the country. Aadhaar Pay
service will enable people to make and receive payments using their Aadhaar number and the
biometric technology, obviating the need to carry mobile phones for payments.

60. With reference to ‘India Post Payment Bank’, consider the following statements:
1. India Post is the first government organisation to start the payment banking service.
2. It can provide maximum ₹1 lakhs as loans to the individuals and small businesses.
3. It is set up as the 100 percent government owned Public Sector Company.
Which of the statements given above is/are correct?
(a) 1 only (b) 2 and 3 only (c) 1 and 3 only (d) 1, 2 and 3

Answer (c)
Explanation
India Post is the third entity to receive the ‘Payment Bank’ licence from the RBI after Airtel and
Paytm. It is the first government organisation to start the payment banking service. It can accept
maximum ₹1 lakhs as deposits from the individuals and small businesses and provides no credit
facility. It is set up as 100 per cent government owned Public Sector Company under the
Department of Posts.

61. Regarding ‘Participatory Notes’ also called P-Notes, which of the following statements is/are correct?
1. These are issued by Foreign Institutional Investors (FIIs).
2. They follow KYC norms.
3. They are a potential source of money laundering in India.
Select the correct answer using the code given below.
(a) 1 only (b) 2 and 3 only (c) 1 and 3 only (d) 1, 2 and 3

Answer (d)
Explanation
P-Notes or Participatory Notes are Overseas Derivative Instruments that have Indian stocks as their
underlying assets. They allow foreign investors to buy stocks listed on Indian exchanges without being
registered. The instrument gained popularity as FIIs, to avoid the formalities of registering and to
remain anonymous, started betting on stocks through this route.
The primary reason why P-Notes are worrying is because of the anonymous nature of the instrument
as these investors could be beyond the reach of Indian regulators. Further, there is a view that it is
being used in money laundering with wealthy Indians, like the promoters of companies, using it to
bring back unaccounted funds and to manipulate their stock prices.
From January 2011, FIIs have had to follow KYC norms and submit details of transactions. In 2014,
new rules on foreign portfolio investors (FPIs) made it mandatory for those issuing P-Notes to submit a
monthly report disclosing their portfolios. This led to a decline in the number of entities issuing P-
Notes. More recently, SEBI mandated that in addition to KYC, the anti-money laundering rules (AML)
will also be applicable to P-Note holders.

62. With reference to commodity market in India, consider the following statements:
1. Commodity markets in India offer both options trading and futures trading.
2. SEBI is the regulator of commodity market in India.
3. India’s first International stock exchange ‘India INX’ provides platform for commodity derivatives
trading.
Which of the statements given above is/are correct?
(a) 1 only (b) 2 and 3 only (c) 1 and 3 only (d) 1, 2 and 3

Answer (b)
Explanation
Securities and Exchange Board of India (SEBI) is the regulator of commodity market in India. In our
country, only futures trading is allowed in the commodity market. India’s first International stock
exchange ‘India INX’ provides a state of the art electronic platform to facilitate trading, clearing and
settlement of the widest range of global benchmark products across all major asset classes, including
securities, equity derivatives, precious metals, base metals, energy, agricultural commodities, interest
rates, currencies etc. from time to time subject to necessary approvals and operating guidelines from
the regulators.


63. Which one of the following best describes the main objective of ‘Multilateral Instrument’ (MLI)?
(a) It is a new multi-lateral agreement between Britain and European Union for post Brexit era.
(b) It is a new multi-lateral agreement by OECD to prevent Base Erosion and Profit Shifting.
(c) It is the multi-lateral agreement signed among the WTO members to facilitate international trade in
goods.
(d) It is the multi-lateral agreement to limit the emission of Hydroflurocarbons (HFC) that contributes
to global warming.

Answer (b)
Explanation
Multilateral Instrument (MLI) is an agreement put out by OECD, the intergovernmental economic
organisation, to stop Base Erosion and Profit Shifting (BEPS), a practice referring to tax avoidance
strategies that exploit gaps and mismatches in tax rules to artificially shift profits to low or no-tax
locations. MLI is thus a framework through which over 100 countries are collaborating to tackle BEPS
or anti-treaty shopping provisions.

64. In India, digital transaction is promoted by


(a) launching the UPI based BHIM app
(b) unveiling Lucky Grahak Yojna and Digi-Dhan Vyapar Yojna
(c) removing the service charge for e-tickets via IRCTC
(d) All of the above

Answer (d)
Explanation
All the above measures are aimed to promote the digital transactions and to achieve a cashless
economy.

65. Consider the following statements regarding World Trade Organization (WTO):
1. As per the 2015 Nairobi Ministerial Conference of WTO, the developed nations have committed to
remove their farm subsidies immediately.
2. The developing countries have given a time period to remove their export subsidies by 2018 in the
same Conference.
3. The food procurement subsidies of the member countries of WTO are calculated based on the
current market prices.
Which of the statements given above is/are correct?
(a) 1 only (b) 2 only
(c) 2 and 3 only (d) 1, 2 and 3
Answer (b)
Explanation
As per the 2015 Nairobi Ministerial Conference of WTO, the developed nations have committed to
remove their export subsidies immediately not their farm subsidies.
The food procurement subsidies of the member countries of WTO are calculated based on the market
prices prevailing during 1986-88.

66. Which of the statements regarding the ‘inverted duty structure’ given below is/are correct?
1. Inverted Duty structure means an input used for the manufacture of a product attracts higher
import duty than the finished product.
2. It makes the manufactured goods uncompetitive against finished product imports in the domestic
market.
Select the correct answer using the code given below.
(a) 1 only (b) 2 only
(c) Both 1 and 2 (d) Neither 1 nor 2
Answer (c)
Explanation
Inverted Duty structure means an input used for the manufacture of a product attracts higher import
duty than the finished product. Eg: Rubber-20%, Tyres-10%.
It makes the manufactured goods uncompetitive against finished product imports in the domestic
market.

67. ‘RESIDEX’, an index for tracking prices of residential properties in India is published by
(a) National Bank for Agriculture and Rural Development
(b) National Housing Bank
(c) Ministry of Housing and Urban Poverty Alleviation


(d) NITI Aayog
Answer (b)
Explanation
National Housing Bank launched RESIDEX for tracking prices of residential properties, in July
2007.
Earlier Residex includes 26 cities and now NHB is planning to add more under it.

68. Consider the following statements:


1. Customs Union is an economic integration where the member countries maintain a common trade
policy with the third parties.
2. Common Market is an economic integration where the member countries adopt a common
currency.
Which of the statements given above is/are correct?
(a) 1 only (b) 2 only
(c) Both 1 and 2 (d) Neither 1 nor 2
Answer (a)
Explanation
In addition to the provisions of the Customs Union, the Common Market allows the free flow of
labour and capital among the member countries. The common currency adoption is seen in the
Economic and Monetary Union.

69. Which of the statement regarding the ‘Dollar index’ is/are correct?
1. The U.S. dollar index (USDX) is a measure of the value of the U.S. dollar relative to the value of a
basket of currencies of the majority of the U.S.'s most significant trading partners.
2. Currently, this index is calculated by factoring in the exchange rates of six major world currencies
the Euro, Chinese Renminbi, Japanese yen, Canadian dollar, British pound and Swiss franc.
Select the correct answer using the code given below.
(a) 1 only (b) 2 only
(c) Both 1 and 2 (d) Neither 1 nor 2
Answer (a)
Explanation
Currently, U.S. dollar index (USDX) is calculated by factoring in the exchange rates of six major
world currencies the euro, Japanese yen, Canadian dollar, British pound, Swedish krona and
Swiss franc.
The euro holds the most weight versus the dollar in the index, constituting about 58% of the
weighting followed by the yen with about 14%. The index started in 1973 with a base of 100, and
values since then are relative to this base.

70. Which of the following statements is correct regarding the Asian Infrastructure Investment Bank
(AIIB)?
(a) India is the third largest shareholder in AIIB after Russia.
(b) The Head Quarters of AIIB is situated in Beijing.
(c) AIIB provides funds to the Asian countries only.
(d) The major non-members are USA, Japan and South Korea.
Answer (b)
Explanation
The Asian Infrastructure Investment Bank (AIIB) is an international financial institution proposed
by China to provide financing for infrastructure projects mainly in Asia. India is the second
largest shareholder after China and the Russia occupies the third position.
According to the Articles of Agreement (AOA) of AIIB, the Bank will “provide or facilitate financing to
any member, or any agency, instrumentality or political subdivision thereof, or any entity or
enterprise operating in the territory of a member, as well as to international or regional agencies or
entities concerned with economic development of the Asia region.”
The major non-members are USA, Japan, Taiwan, Canada etc.

71. Consider the following statements regarding the ‘Junk bonds’, sometimes seen in the news recently:
1. These are high yield bonds.
2. They have low credit rating by the rating agencies.
3. They are low risky instruments.
Which of the statements given above is/are correct?
(a) 1 and 2 only (b) 2 and 3 only
(c) 3 only (d) 1 and 3 only


Answer (a)
Explanation
A junk bond refers to high-yield or noninvestment-grade bonds. Junk bonds are fixed-income
instruments that carry a credit rating of BB or lower by Standard & Poor's, or Ba or below
by Moody's Investors Service. Junk bonds are so called because of their higher default risk in
relation to investment-grade bonds.

72. Which of the following is/are the likely reasons for the low public acceptance of the ‘Gold Monetisation
Scheme’ unveiled by the Government of India?
(a) The owners of the gold must shoulder the cost of testing its purity and melting it down.
(b) The return from the scheme is well below the bank deposits
(c) The non-participation of banks owing less returns
(d) All of the above
Answer (d)
Explanation
All of the options given in the question are true for the lack of enthusiasm in the GMS by both
customers and associated institutions.

73. Consider the following statements, regarding the Geographical Indication (GI) status:
1. GI status is a community right.
2. GI status can be licensed to others.
3. GI status is given to domestic as well as foreign products.
Which of the statements above is/are correct?
(a) 1 only (b) 2 and 3 only
(c) 1 and 3 only (d) 1, 2 and 3
Answer (c)
Explanation
Being a community right, GI can’t be licensed to others. GI is accorded not only to the domestic
products but also to the foreign products. India has given its GI recognition to countries like Peru,
USA, UK, Italy, France etc. Peruvian Pisco is the first foreign product to receive the GI tag in India.

74. Which of the following reports is/are published by the World Bank?
1. Universal Healthcare Index
2. Doing Business Report
3. World Economic Outlook
4. Global Financial Development Report
Select the correct answer using the code given below.
(a) 1 only (b) 2 and 3 only
(c) 1 and 3 only (d) 1, 2 and 4 only
Answer (d)
Explanation
The major reports published by World Bank are
I. Global Financial Development Report
II. Global Economic Prospectus
III. Global Monitoring Report
IV. World Development Report
V. Doing Business Report
VI. International Debt Statistics
VII. Universal Healthcare Index
The major reports of IMF are
I. World Economic Outlook
II. Fiscal Monitor
III. Global Financial Stability Report

75. Consider the following statements:


1. In WTO, all the decisions are taken on the basis of consensus.
2. WTO takes measures to protect not only the environment but also the public health, animal health
and plant health.
3. It assists the developing countries in trade policy issues, through financial as well as technical
assistance and training programmes.
Which of the statements given above is/are correct?
(a) 1 and 2 only (b) 2 and 3 only
(c) 3 only (d) 1 and 3 only

Answer (a)
Explanation
There is no financial support from the World Trade Organization to their member countries.

76. Consider the following statements regarding ‘Aadhaar Pay’:


1. It is a digital platform unveiled by the NITI Aayog to ease the digital transactions for merchants.
2. RuPay cards are necessary to carry out the transactions under it.
3. Merchants have to pay a certain amount as transaction charge to banks.
Which of the statements given above is/are correct?
(a) 1 and 2 only (b) 2 and 3 only
(c) 3 only (d) 1 and 3 only

Answer (c)
Explanation
Aadhaar Pay is a digital payment platform for merchants and is meant to receive digital
payments from customers over the counter through Aadhaar authentication. National Payments
Corporation of India (NPCI), the implementing agency for Aadhaar Enabled Payments Systems
(AEPS) is the agency behind the Aadhaar Pay.
Aadhaar Pay app enables consumers to pay without any physical payment instrument. . It will be
specifically beneficial for those who do not have debit cards, mobile wallets and mobile phones.
For Aadhaar Pay transactions the merchants will have to pay to their respective banks the
merchant discount rate, or MDR, which has been fixed at 0.25% of the transaction amount.

77. Consider the following statements regarding the sovereign credit rating of India:
1. The credit rating of India has been remaining unchanged for last several years at the lowest
investment grade.
2. Organisation for Economic Cooperation and Development has been responsible for revising the
credit rating of various countries.
Which of the statements given above is/are correct?
(a) 1 only (b) 2 only
(c) Both 1 and 2 (d) Neither 1 nor 2
Answer (a)
Explanation
The credit rating of India has been remaining unchanged for last several years at BBB- or the lowest
investment grade which is only a grade above the junk status for the government/sovereign bonds.
Leading international credit rating agencies like Standard & Poor’s, Moody’s, Fitch etc. rate the
government/sovereign bonds on the basis of lending risks.

78. Consider the following statements regarding the ‘Sovereign Gold Bond’ (SGB) scheme.
1. SGBs are government securities denominated in grams of gold.
2. Investors have to pay the issue price in cash and the bonds will be redeemed in cash on maturity.
3. Bonds are sold through scheduled commercial banks (excluding RRBs), Share Holding Corporation
of India Ltd. offices and designated Post Offices either directly or through their agents.
Which of the statements given above is/are correct?
(a) 1 only (b) 1 and 2 only
(c) 2 and 3 only (d) 1, 2 and 3
Answer (d)
Explanation
SGBs are government securities denominated in grams of gold. They are substitutes for holding
physical gold. Investors have to pay the issue price in cash and the bonds will be redeemed in cash
on maturity. The Bond is issued by Reserve Bank on behalf of Government of India. The Bonds
are issued in denominations of one gram of gold and in multiples thereof. Minimum investment in
the Bond shall be one gram with a maximum buying limit of 500 grams per person per fiscal
year (April – March). In case of joint holding, the limit applies to the first applicant.

79. Consider the following statements regarding the ‘Goods and Service Tax’:
1. GST proposes a four tax slabs at 5%, 12%, 18% and 28%.
2. A minimum of 15% cess on top of the peak GST rate will be levied on the luxury goods and aerated
drinks.
3. All the Union Territories except with that of state legislatures are governed by the Central GST
laws.
Which of the statements given above is/are correct?
(a) 1 only (b) 1 and 2 only
(c) 2 and 3 only (d) 1, 2 and 3


Answer (a)
Explanation
‘Goods and Service Tax’ is one of the major economic reforms in the indirect tax system of India after
independence. GST proposes ‘One Nation, One Tax, Four Rates’ concept with the tax slabs as 5%,
12%, 18% and 28%.
Goods and Service Tax Council, the highest decision making body in case of GST, has decided
that a maximum of 15% cess on top of the peak GST rate 28 % will be levied on the luxury goods
and aerated drinks.
The State GST draft law will have to be approved by the legislative assemblies of states and
union territories (Delhi, Puducherry). The Union Territories GST draft law is for the union
territories like Andaman and Nicobar Islands, Lakshadweep, Daman and Diu and Dadra and Nagar
Haveli, which do not have legislative assemblies.

80. Consider the following statements regarding the Sovereign Gold Bond (SGB) scheme.
1. The minimum investment in the SGB is 1 gram with no upper limit.
2. The tenure of the bond is 8 years with exit option available from 5 th year onwards.
3. Bonds can be used as collateral for the loans.
Which of the statements given above is/are correct?
(a) 1 only (b) 2 and 3 only
(c) 1 and 3 only (d) 1, 2 and 3
Answer (b)
Explanation
Sovereign Gold Bonds will be issued on payment of rupees and denominated in grams of gold. The
Bonds are issued by the Reserve Bank of India on behalf of the Government of India. The bonds
are distributed through banks, designated post offices, Stock Holding Corporation of India and
recognised stock exchanges- BSE and NSE.
Minimum investment in the bond shall be 1 gram. The bonds can be bought by Indian residents or
entities and is capped at 500 grams. The tenor of the bond is for a minimum of 8 years with option
to exit in 5th, 6th and 7th years. Bonds can be used as collateral for loans. Capital gain tax
arising on redemption of SGB to an individual has been exempted.

81. How the Trade Facilitation Agreement in Goods by the WTO improves the global economy?
1. The participant nations will simplify and standardise the custom procedures at the borders to
make easier flow of goods around the globe.
2. It will trim the global trade cost by more than 50 percent and could boost global growth by a half a
percentage point per year.
3. Poorer countries get improved access to richer market.
Select the correct answer using the code given below.
(a) 1 only (b) 2 and 3 only
(c) 1 and 3 only (d) 1, 2 and 3
Answer (c)
Explanation
Trade Facilitation Agreement (TFA) in Goods is the first multilateral deal agreed at the World
Trade Organization (WTO) after it came into force. Under the deal, nations agreed to simplify and
standardise customs procedures at borders to make it easier for goods to flow around the world.
TFA was estimated to trim global trade costs by more than 14% and could boost global growth by
half a percentage point per year. Poorer countries are expected to reap the most benefits from the
TFA through provisions that will improve access to richer markets for their products.

82. Which of the following are the measures taken by the Government of India to curb black money in the
economic system?
1. The information exchange agreement with Switzerland
2. The Benami Transactions Act, 2016.
3. Demonetisation of higher denomination currencies.
4. Setting up Special Investigation team.
Select the correct answer using the code given below.
(a) 1, 2 and 3 only (b) 2 and 4 only
(c) 1 and 3 only (d) 1, 2, 3 and 4
Answer (d)
Explanation
Demonetisation is the part of the Government’s resolve to eliminate corruption, black money,
counterfeit currency and terror funding. The demonetisation has strong potential to generate long-
term benefits in terms of greater digitisation of the economy, increased flow of financial savings


and greater formalisation of the economy, all of which would eventually lead to higher GDP
growth and tax revenues.
The action followed a series of earlier efforts to curb such illicit activities, including the creation of
the Special Investigation Team (SIT) in the 2014 budget, the Black Money Act, 2015; the
Benami Transactions Act of 2016; the information exchange agreement with Switzerland,
changes in the tax treaties with Mauritius and Cyprus, and the Income Disclosure Scheme.

83. In the context of which of the following do you find the term ‘bancassurance’ in the news?
(a) Banking affairs
(b) Insurance Sector
(c) Small saving schemes
(d) Both (a) and (b)
Answer (d)
Explanation
Bancassurance means selling insurance product through banks. Banks and insurance company come
up in a partnership wherein the bank sells the tied insurance company's insurance products to its
clients.

84. Which of the following is/are the recommendations of the Fiscal Responsibility and Budgetary
Management (FRBM) committee by NK Singh?
1. Setting up of the Fiscal Council to oversee and verify the credibility of the government’s budget
numbers and fiscal target for any year.
2. Setting a roadmap to achieve a fiscal deficit of 2.5 per cent of gross domestic product by 2023.
3. Recommending a fiscal deficit target of 3.2 percent of gross domestic product for financial year
2017-18 and 3 percent for financial years 2018-19 and 2019-20.
Select the correct answer from the code given below.
(a) 1 and 2 only (b) 2 and 3 only
(c) 3 only (d) 1 and 3 only
Answer (a)
Explanation
The NK Singh committee recommends a fiscal deficit of 3 percent of gross domestic product for
the financial years from 2017-18 to 2019-20. But in the budget of 2017-18, the Union Government
preferred to keep the fiscal deficit of 3.2 percent of GDP and to follow the 3 percent limit for next
couple of financial years.

85. Securities and Exchange Board of India (SEBI) regulates trading in which of the following markets?
1. Stock market
2. Commodity futures market
3. Bullion market
Select the correct answer using the code given below.
(a) 1 and 2 only (b) 2 and 3 only
(c) 3 only (d) 1, 2 and 3
Answer (a)
Explanation
Securities and Exchange Board of India (SEBI) regulates the trading in both stock exchanges as well
as commodity future markets. There is no specific regulator for the bullion trading in India.

86. Which one of the following best describes the term ‘import alert’, sometimes seen in the news?
(a) It is the warning issued by the Ministry of Commerce & Industry in India against the excess import
of particular commodity into the market.
(b) It is the warning issued by the EU against the products affected by the disease causing organisms.
(c) It is the warning issued by the US Food and Drug administration against the imported
pharmaceutical products that do not meet their quality criteria.
(d) None of the above statements (a), (b) and (c) are correct in this context.
Answer (c)
Explanation
The United States Food and Drug Administration (USFDA) has recently issued an import alert for
Divi’s Laboratories’ Visakhapatnam unit-II due to quality and data integrity issues. The company is
engaged in manufacturing active pharmaceutical ingredients (APIs) and intermediates for generics,
custom synthesis of API’s and advanced intermediates for discovery compounds for pharma giants.
An import alert means that products from the facility, which are meant for imports, can be
refused without examination and will not be allowed to be marketed in the US.


87. Which of the following entity has not received the payment bank licence in India?
(a) Airtel
(b) India Post
(c) Capital Local Area Bank Ltd.
(d) Paytm
Answer (c)
Explanation
Capital Local Area Bank Ltd. has received the licence for ‘Small Finance Bank’ from RBI and it has
started its operation as the first Small Finance Bank in India.

88. Which of the following statements regarding the BRICS led New Development Bank is not correct?
(a) Its headquarters is located at Shanghai.
(b) Its first President is from India
(c) Its first regional office will be set up in Johannesburg, South Africa.
(d) China holds the maximum shareholding in NDB.
Answer (d)
Explanation
All the BRICS countries hold equal share in the NDB.

89. Which of the following statements regarding the Foreign Investment Promotion Board is correct?
(a) It is chaired by the Minister of Finance, Govt. of India
(b) It is responsible for the approval of any amount of foreign investment coming into India.
(c) It gives its nod for foreign investment coming into India via automatic route only.
(d) None of the statement (a), (b) and (c) is correct in this context.
Answer (d)
Explanation
The Foreign Investment Promotion Board (FIPB), housed in the Department of Economic Affairs,
Ministry of Finance, is an inter-ministerial body, responsible for processing of FDI proposals and
making recommendations for Government approval.
FIPB comprises of the following Secretaries to the Government of India:
(i) Secretary to Government, Department of Economic Affairs, Ministry of Finance – Chairperson
(ii) Secretary to Government, Department of Industrial Policy & Promotion, Ministry of Commerce &
Industry
(iii) Secretary to Government, Department of Commerce, Ministry of Commerce & Industry
(iv)Secretary to Government, Economic Relations, Ministry of External Affairs
(v) Secretary to Government, Ministry of Overseas Indian Affairs.
FIPB can give nod upto FDI cap worth ₹5000 Crore under the approval route.

90. MUDRA (Micro Units Development & Refinance Agency Ltd) is a financial institution being set up by
Government of India for development and refinancing micro units’ enterprises. Consider the following
statements regarding the same.
1. MUDRA provides three types of loans namely Shishu, Kishor and Tarun.
2. Any loan by a financial institution to Small and Medium Enterprises on or after 08 April 2015,
above ₹10 lakh is termed as Mudra loan.
3. ‘Activities allied to agriculture’ also included under the Pradhan Mantri Mudra Yojna.
Which of the statements given above is/are correct?
(a) 1 and 2 only (b) 2 and 3 only
(c) 3 only (d) 1 and 3 only
Answer (d)
Explanation
Under the MUDRA, there are 3 types of loans are there.
• Shishu : covering loans up to ₹ 50,000/-
• Kishor : covering loans above ₹ 50,000/- and up to ₹ 5 lakh
• Tarun : covering loans above ₹ 5 lakh and up to ₹ 10 lakh
Department of Financial Services (DFS) vide its letter dated May 14, 2015 have advised all PSBs, RRBs
and Scheduled Cooperative Banks regarding roll out of PMMY and to cover all loans up to loan size
of 10 lakh sanctioned on or after April 08, 2015 for income generation under PMMY.
Following is an illustrative list of the activities that can be covered under MUDRA loans:
1 Transport Vehicle
2 Community, Social & Personal Service Activities
3 Food Products Sector
4 Textile Products Sector / Activity


5 Business loans for Traders and Shopkeepers
6 Equipment Finance Scheme for Micro Units
7 Activities allied to agriculture

91. Consider the following pairs:


1. Mode 1 service : Consumption abroad
2. Mode 2 service : Commercial presence
3. Mode 4 service : Movement of natural persons
Which of them is/are correctly matched?
(a) 1 and 2 only (b) 2 and 3 only
(c) 3 only (d) 1 and 3 only
Answer (c)
Explanation
The definition of services trade under the GATS is four-pronged, depending on the territorial presence
of the supplier and the consumer at the time of the transaction
a. from the territory of one Member into the territory of any other Member (Mode 1 — Cross border
trade);
b. in the territory of one Member to the service consumer of any other Member (Mode 2 —
Consumption abroad);
c. by a service supplier of one Member, through commercial presence, in the territory of any other
Member (Mode 3 — Commercial presence); and
d. by a service supplier of one Member, through the presence of natural persons of a Member in the
territory of any other Member (Mode 4 — Presence of natural persons).

92. Which of the following statements regarding the PM Garib Kalyan Yojna is/are correct?
1. It is carried out by the Income Tax Department, Government of India.
2. Under this scheme, total penalty amounts to 25% of the total income declared and there is a
mandatory deposit of 50% of the total income in PMGKY Deposit scheme for four years.
Select the correct answer using the code given below.
(a) 1 only (b) 2 only
(c) Both 1 and 2 (d) Neither 1 nor 2
Answer (a)
Explanation
PMGKY is Union Government’s second income disclosure scheme (IDS) to allow tax evaders to come
clean with unaccounted wealth. It provides for 50 per cent tax and surcharge on declarations of
unaccounted cash deposited in banks.
Declarant of undisclosed income needs to pay 30% tax, 10% penalty and 33% Pradhan Mantri Garib
Kalyan Cess on the tax, all of which add up to around 50%. Besides, declarant must make mandatory
deposit of 25% of undisclosed income in the zero-interest Pradhan Mantri Garib Kalyan Deposit
Scheme, 2016 with lock-in period of 4 years.

93. Which of the following the provisions are covered under the model Bilateral Investment Treaty
proposed by Government of India?
1. Most Favoured Nation concept
2. Mandatory litigation in domestic courts
3. Setting up multilateral investment courts
Select the correct answer using the code given below.
(a) 1 and 2 only (b) 2 only
(c) 3 only (d) 1 and 3 only
Answer (b)
Explanation
Rattled by many BIT claims brought by foreign investors from 2011 onwards, last year, India
unilaterally issued BIT termination notices to 58-member countries. Reportedly, these BITs would
lapse on March 31 after the expiry of the mandatory one-year notice period.
But most developed countries have not shown much interest in the Model BIT that India proposed
because of the following provisions.
India’s model BIT excludes Most Favoured Nation (MFN) provision — a cornerstone of non-
discrimination in international economic relations. Second, the Indian model completely excludes
taxation from the purview of the BIT — a direct response to Vodafone and Cairn Energy bringing BIT
claims against India for imposing taxes retrospectively. Third, the Indian model completely excludes
issuance of compulsory licenses (CLs) and revocation of intellectual property rights (IPR) from its
purview. The Indian model BIT, also mandatorily requires foreign investors to litigate in domestic


courts for five years before pursuing a claim under international law which is not so attractive for
foreign firms.

94. Which of the following banks will be merged with the State Bank of India by the start of financial year
2017-18?
1. Bharatiya Mahila Bank
2. State Bank of Patiala
3. State Bank of Travancore
4. State Bank of Bikaner and Jaipur
Select the correct answer using the code given below.
(a) 1 and 4 only (b) 2, 3 and 4 only
(c) 1, 2 and 3 only (d) 1, 2, 3 and 4

Answer (d)
Explanation
The Government of India has set the date for the record merger of the State Bank of India with its
five associate banks on April 1, 2017. The five associate banks are the State Bank of Bikaner and
Jaipur (SBBJ), the State Bank of Mysore (SBM), the State Bank of Travancore (SBT), the State
Bank of Hyderabad (SBH) and the State Bank of Patiala (SBP). Bharatiya Mahila Bank (BMB) will
also merged with SBI on April 1, 2017.

95. Consider the following statements regarding the Liquidity Adjustment Facility:
1. It is one of the chief monetary tools of the RBI.
2. Under LAF, RBI sells Government Securities when there are inflationary trends in the economy and
buys the same during deflationary periods.
Which of the above statements is/are correct?
(a) 1 only (b) 2 only
(c) Both 1 and 2 (d) Neither 1 nor 2
Answer (a)
Explanation
LAF is a tool used in monetary policy that allows banks to borrow money through repurchase
agreements. This arrangement allows banks to respond to liquidity pressures and is used by
governments to assure basic stability in the financial markets. It consists of repo and reverses repo
operations.
Buying and selling of the government securities are part of the Open Market Operations.

96. Consider the following statements regarding the Central Public Sector Enterprises-Exchange Traded
Funds (CPSE-ETF):
1. CPSE-ETF follows the Nifty50 index.
2. It includes Maharatna, Navaratna as well as Miniratna companies.
3. ONGC holds the maximum share of the CPSE ETF.
Which of the above statements is/are correct?
(a) 1 and 2 only (b) 2 and 3 only
(c) 3 only (d) 1 and 3 only
Answer (c)
Explanation
Central Public Sector Enterprises (CPSE) ETF, which tracks the Nifty CPSE Index, is a
concentrated portfolio of 10 stocks, whose main aim is to help the Government of India (GOI) in
disinvesting its stake in a few CPSEs via the ETF route. The portfolio comprises of 10 Maharatna and
Navaratna CPSEs whose weightage in the index are given below.


97. Which of the following is/are the major constraints for the better monetary transmission in the Indian
economy?
1. Deposit rates of the small savings schemes are higher compared to the bank deposits.
2. Twin Balance Sheet Problem.
3. High Statutory Reserve Requirements.
Select the correct answer using the code given below.
(a) 2 and 3 only (b) 1 and 2 only
(c) 1 and 3 only (d) 1, 2 and 3
Answer (d)
Explanation
The Indian banking system suffers from the inefficient monetary transmission due to following
reasons:
1. Deposit rates of the small savings schemes are higher compared to the bank deposits.
Banks find it more difficult to cut lending rates immediately after a cut in the policy rate because the
cost of deposits does not adjust commensurately and immediately, given the fixed nature of deposit
contracts. However, banks can raise lending rates far more quickly after a policy rate hike, because
loans are mostly at variable rates, and can be re-priced faster. There is a competitive constraint of
reducing deposit rates too much and too quick as well. If bank deposit rates are lower than small
savings rates which are administratively fixed, there could be some deposit flight to small savings
schemes providing higher returns.
2. Twin balance sheet problem
The banking system continues to be under stress due to rising non-performing assets (NPAs), which
along with restructured assets are estimated at about 10 per cent of total assets currently. The
stressed loans are concentrated across five key sectors - infrastructure, iron and steel, textiles,
aviation and mining - which constitute 24 per cent of total advances and 52 per cent of total stressed
advances. Given this backdrop, it is natural to expect banks to be risk-averse and show reluctance in
lending further to these sectors.
3. High Statutory Reserve Requirements
Some commercial banks have argued that a cash reserve ratio (CRR) cut is required to make the
monetary transmission effective. Currently, commercial banks have to maintain four per
cent CRR with RBI, on which they do not earn any interest. A meaningful cut in the CRR will not only
help inject additional liquidity into the system, thereby freeing resources to lend, but also
help banks to pass on rate cuts without taking a hit on their net interest margin.

98. Consider the following statements regarding the Repo rate:


1. It is the rate at which the RBI lends money to commercial banks generally against the Government
Securities.
2. It is the rate at which the RBI borrows money from the commercial banks against the Government
Securities.
3. It is the rate at which the Commercial Banks keep deposits with the RBI.
Which of the above statements is/are correct?
(a) 1 only (b) 2 only
(c) 2 and 3 only (d) None of the above
Answer (a)
Explanation
Under the Repo or Repurchase Option the banks borrow money from the RBI to meet the short
term needs by selling the securities to RBI with an agreement to repurchase the same at the
predetermined rate and date. The rate charged by the RBI is called the repo rate.

99. Which of the following criteria is/are considered while computing the Multi-Dimensional Poverty Index
of UN Development Programme?
1. Nutrition
2. Expected years of schooling
3. Standard of living
4. Child Immunisation
Select the correct answer using the code given below.
(a) 2 and 4 (b) 1, 2 and 3
(c) 1, 2 and 4 (d) 3 and 4
Answer (b)
Explanation
The Human Development Index (HDI) is a summary measure of average achievement in key
dimensions of human development: a long and healthy life, being knowledgeable and have a


decent standard of living. The HDI is the geometric mean of normalized indices for each of the
three dimensions.
The health dimension is assessed by life expectancy at birth; the education dimension is measured by
mean of years of schooling for adults aged 25 years and more and expected years of schooling for
children of school entering age. The standard of living dimension is measured by gross national income
per capita. The HDI uses the logarithm of income, to reflect the diminishing importance of income with
increasing GNI. The scores for the three HDI dimension indices are then aggregated into a composite
index using geometric mean.

The Multidimensional Poverty Index (MPI), published for the first time in the 2010 Report,
complements monetary measures of poverty by considering overlapping deprivations suffered by
individuals at the same time. The index identifies deprivations across the same three dimensions as
the HDI and shows the number of people who are multidimensionally poor (suffering deprivations in
33% or more of the weighted indicators) and the number of weighted deprivations with which poor
households typically contend with. About 1.5 billion people in the 102 developing countries
currently covered by the MPI—about 29 percent of their population — live in multidimensional
poverty.

100. Which of the following statements regarding the 2016 Human Development Report (HDR) released by
the United Nations Development Programme (UNDP) is/are correct?
1. Both India and Pakistan comes under the ‘medium development’ category.
2. Among SAARC members, India is fourth after Sri Lanka, Maldives and Bhutan.
3. While India’s HDI value increased from 0.428 in 1990 to 0.624 in 2015, it still had the lowest rank
among BRICS nations.
Select the correct answer using the code given below.
(a) 1 and 2 only (b) 1 and 3 only
(c) 2 and 3 only (d) 1, 2 and 3
Answer (b)
Explanation
India slipped down one place from 130 to 131 among the 188 countries ranked in terms of human
development, says the 2016 Human Development Report (HDR) released by the United Nations
Development Programme (UNDP). India’s human development index (HDI) value of 0.624 puts it in
the “medium human development” category, alongside countries such as Congo, Namibia and
Pakistan. It is ranked third among the SAARC countries, behind Sri Lanka (73) and the Maldives
(105), both of which figure in the “high human development” category.
While India’s HDI value increased from 0.428 in 1990 to 0.624 in 2015, it still had the lowest rank
among BRICS nations. However, its average annual growth in HDI (1990-2015) was higher than that
of other medium HD countries.

101. Which of the following items is/are exempted under the Goods and Services Tax system?
1. Electricity
2. Real Estate
3. Petroleum products
4. Alcohol
Select the correct answer using the code given below.
(a) 1, 2, 3 and 4 (b) 2 and 4 only
(c) 1 and 3 only (d) 1, 2 and 3 only

Answer (a)
Explanation
Goods and Service Tax is a major economic reform in India after the 1991 economic liberalisation.
GST, proposed to be rolled out from 1st July 2017, will have a four tier structure with rates ranging
from 5% to 28%. But regrettably, a large chunk of the economy, which includes real estate, electricity,
alcohol and petroleum products, is out of GST.

102. Which of the following taxes is/are subsumed under the proposed Goods and Services Tax?
1. Entry Tax
2. Service Tax
3. Customs Duty
4. Countervailing Duty
Select the correct answer using the code given below.
(a) 1, 2 and 3 only (b) 1, 2 and 4 only
(c) 2, 3 and 4 only (d) 1, 3 and 4 only
Answer (b)
Explanation
At the Central level, the following taxes are being subsumed:
(a) Central Excise Duty,
(b) Additional Excise Duty,
(c) Service Tax,
(d) Additional Customs Duty commonly known as Countervailing Duty, and
(e) Special Additional Duty of Customs.
At the State level, the following taxes are being subsumed:
(a) Subsuming of State Value Added Tax/Sales Tax,
(b) Entertainment Tax (other than the tax levied by the local bodies), Central Sales Tax (levied by
the Centre and collected by the States),
(c) Octroi and Entry tax,
(d) Purchase Tax,
(e) Luxury tax, and
(f) Taxes on lottery, betting and gambling.

103. Consider the following statements regarding the Monetary Policy Committee (MPC) of RBI:
1. MPC includes 6 members and is headed by RBI Governor.
2. MPC is taking decisions based on consensus.
3. MPC has only advisory power on monetary policy and the policy rate is set by Government of India.
4. MPC members appointed by the Central Government hold the office for a period of four years and
also eligible for reappointment.
Which of the statements given above is/are correct?
(a) 1 only (b) 1 and 4 only
(c) 1 and 3 only (d) 2 and 4 only
Answer (a)
Explanation
The Reserve Bank of India Act, 1934 (RBI Act) has been amended by the Finance Act, 2016, to
provide for a statutory and institutionalised framework for a Monetary Policy Committee, for
maintaining price stability, while keeping in mind the objective of growth. The Monetary Policy
Committee would be entrusted with the task of fixing the benchmark policy rate (repo rate)
required to contain inflation within the specified target level.
As per the provisions of the RBI Act, out of the six Members of Monetary Policy Committee, three
Members will be from the RBI and the other three Members of MPC will be appointed by the
Central Government. The committee is headed by the Governor of the RBI. The Members of the
Monetary Policy Committee appointed by the Central Government shall hold office for a period of
four years and there will not be eligible for reappointment.
Decisions in MPC are taken based on voting with each member having a single vote and the
Chairman having casting vote in case of tie.

104. Which of the following statements regarding the Index of Industrial Production (IIP) is correct?
(a) IIP is published by the Central Statistical Organisation
(b) IIP gives maximum weightage to the manufacturing sector
(c) IIP receives data compilation from 16 different agencies across the country
(d) All the statements (a), (b) and (c) are correct in this context.


Answer (d)
Explanation
The Index of Industrial Production (IIP) is an index which shows the growth rates in different
industry groups of the economy in a stipulated period of time. The IIP index is computed and
published by the Central Statistical Organisation (CSO) on a monthly basis. Currently IIP figures are
calculated considering 2004-05 as base year.
IIP data is based on the 3 sectors viz. mining, manufacturing and electricity with maximum
weightage to manufacturing sector. The data compilation is received from the 16 different agencies
across the country.

105. Which of the statements regarding the copyright is not correct?


(a) Copyright protection extends to expressions, ideas, procedures, mathematical concepts etc.
(b) Copyright protects both economic and moral rights.
(c) The economic rights under the copyright have a time limit.
(d) None of the above
Answer (a)
Explanation
Copyright (or author’s right) is a legal term used to describe the rights that creators have over
their literary and artistic works. Works covered by copyright range from books, music, paintings,
sculpture, and films, to computer programs, databases, advertisements, maps, and technical
drawings.
Works commonly protected by copyright throughout the world include:
• literary works such as novels, poems, plays, reference works, newspaper articles;
• computer programs, databases;
• films, musical compositions, and choreography;
• artistic works such as paintings, drawings, photographs, and sculpture;
• architecture; and
• Advertisements, maps, and technical drawings.
Copyright protection extends only to expressions, and not to ideas, procedures, methods of
operation or mathematical concepts as such. Copyright may or may not be available for a number
of objects such as titles, slogans, or logos, depending on whether they contain sufficient authorship.
There are two types of rights under copyright:
• economic rights, which allow the rights owner to derive financial reward from the use of his works
by others; and
• moral rights, which protect the non-economic interests of the author.
Economic rights have a time limit, which can vary according to national law. In those countries
which are members of the Berne Convention, the time limit should be equal to or longer than 50
years after the creator’s death. Longer periods of protection may however be provided at the national
level.

106. Consider the following statements:


1. Non-convertible debentures are used as tools to raise long-term funds by companies through a
public issue.
2. Non-convertible debentures can't be converted into shares or equities after sometime.
Which of the statements given above is/are correct?
(a) 1 only (b) 2 only
(c) Both 1 and 2 (d) Neither 1 nor 2
Answer (c)
Explanation
Debentures are long-term financial instruments which acknowledge a debt obligation towards the
issuer. Some debentures have a feature of convertibility into shares after a certain point of time at
the discretion of the owner. The debentures which can't be converted into shares or equities are
called non-convertible debentures (or NCDs).
Non-convertible debentures are used as tools to raise long-term funds by companies through a
public issue. To compensate for this drawback of non-convertibility, lenders are usually given a
higher rate of return compared to convertible debentures.

107. Which of the following is the major objective behind the proposed ‘Standing Deposit Facility’ under
RBI?
(a) To help the Reserve Bank of India absorb surplus funds without having to provide lenders
collateral in exchange.
(b) To help the Reserve Bank of India inject sufficient capital into the economy during demonetisation
period.


(c) To help the Government of India raise the funds for farm loan waiver schemes.
(d) To help the banks borrow from the Reserve Bank of India in an emergency situation when inter-
bank liquidity dries up completely.
Answer (a)
Explanation
India is planning to introduce a new monetary-policy tool in the financial year 2017-18 to better
manage a banking system swimming in excess cash. The so-called Standing Deposit Facility, or SDF,
will help the Reserve Bank of India absorb surplus funds without having to provide lenders
collateral in exchange. In 2014, a panel led by RBI Governor Urjit Patel -- who was at that time a
deputy -- had proposed the introduction of the SDF as part of measures to improve the monetary-
policy framework.

108. Which of the statement regarding the Banks Board Bureau (BBB) is not correct?
(a) BBB will recommend selection of heads for Public Sector Banks
(b) BBB is chaired by the RBI Governor
(c) BBB is based in Mumbai
(d) BBB is an autonomous body
Answer (b)
Explanation
With a view to improve the governance of Public Sector Banks (PSBs), the Government of India has
set up an autonomous Banks Board Bureau(BBB). The Bureau will recommend for selection of
heads - Public Sector Banks and Financial Institutions (even from the private sector, if need be) and
help banks in developing strategies and capital raising plans. It will also advise banks how to go
ahead with mergers and acquisitions. Banks Board Bureau has three ex-officio members and
three expert members in addition to Chairman. All the Members and Chairman will be part time.
The appointments have been made for a period of two years.
BBB, which came to life in 2016, as part of the seven-point Indradhanush plan will constantly
engage with the boards of all 22 public sector banks to formulate appropriate strategies for their
growth and development. It would also be responsible for selection of non-executive chairman and
non-official directors on the boards. Presently BBB is chaired by the former CAG Vinod Rai.

109. Which of the following pairs is/are correctly matched?


Economic Funds associated
activity/Economic units with them
1. Disinvestment : NIF
2. Infrastructure : NIIF
Development
3. CPSE : ETF
Select the correct answer using the code given below.
(a) 1 and 2 only (b) 2 only
(c) 3 only (d) 1, 2 and 3
Answer (d)
Explanation
National Investment and Infrastructure Fund (NIIF) is a major initiative by the Government of India
to give fillip to Infrastructure sector and spur economic growth. Its creation was announced in
the Union Budget 2015-16. The objective of NIIF would be to maximize economic impact mainly
through infrastructure development in commercially viable projects, both green field and brown
field, including stalled projects. It could also consider other nationally important projects, for
example, in manufacturing, if commercially viable.
The Cabinet Committee on Economic Affairs (CCEA) on 27th January, 2005 had approved the
constitution of a National Investment Fund (NIF). The Purpose of the fund was to receive
disinvestment proceeds of central public sector enterprises and to invest the same to generate
earnings without depleting the corpus. The earnings of the Fund were to be used for
selected Central social welfare Schemes. This fund was kept outside the consolidated fund of India.
Exchange Traded Funds (ETFs), as their name denotes, are mutual funds that are traded on stock
exchanges and bought and sold like stocks. They mostly track an underlying index and are passive
investment tools, that is, the fund manager has no role to play in ETFs. The expense ratio is,
therefore, much lower in ETFs.
On 14 March 2017, the Government of India launched the third tranche of Central Public Sector
Enterprises Exchange Traded Fund (CPSE-ETF) worth ` 2,500 crore. It was operated by Reliance
Mutual Fund. The offering was subscribed nearly four times. CPSE-ETF, which tracks the Nifty
CPSE Index, is a concentrated portfolio of 10 stocks (stocks of both Navaratna and Maharatna


companies), whose main aim is to help the Government of India (GOI) in disinvesting its stake in a
few CPSEs via the ETF route. There is an upfront discount of 5 per cent on the Further Fund Offer
(FFO) Reference Market Price to the stocks included in the index.

110. Which of the following public sector organizations are having the ‘Maharatna’ status?
1. Steel Authority of India
2. Indian Railway
3. National Thermal Power Corporation
4. Air India
Select the correct answer using the code given below.
(a) 1 only (b) 2 and 3
(c) 1 and 3 (d) 1, 2 and 4
Answer (c)
Explanation
The eligibility criteria laid down by the Government for grant of Maharatna, Navratna and Miniratna
status to Central Public Sector Enterprises (CPSEs) are following:
Criteria for grant of Maharatna status:-
The CPSEs fulfilling the following criteria are eligible to be considered for grant of Maharatna status.
(i) Having Navratna status.
(ii) Listed on Indian stock exchange with minimum prescribed public shareholding under SEBI
regulations.
(iii) Average annual turnover of more than Rs. 25,000 crore, during the last 3 years.
(iv) Average annual net worth of more than Rs. 15,000 crore, during the last 3 years.
(v) Average annual net profit after tax of more than Rs. 5,000 crore, during the last 3 years.
(vi) Should have significant global presence/international operations.
Criteria for grant of Navratna status:-
The Miniratna Category – I and Schedule ‘A’ CPSEs, which have obtained ‘excellent’ or ‘very good’
rating under the Memorandum of Understanding system in three of the last five years, and have
composite score of 60 or above in the six selected performance parameters, namely,
(i) net profit to net worth,
(ii) manpower cost to total cost of production/services,
(iii) profit before depreciation, interest and taxes to capital employed,
(iv) profit before interest and taxes to turnover,
(v) earning per share and
(vi) inter-sectoral performance.
Criteria for grant of Miniratna status:-
The CPSEs which have made profits in the last three years continuously and have positive net
worth are eligible to be considered for grant of Miniratna status.
Presently, there are 7 Maharatna, 16 Navratna and 71 Miniratna CPSEs.
7 Maharatna companies are:
➢ Steel Authority of India Ltd.
➢ Oil and Natural Gas Corporation
➢ National Thermal Power Corporation
➢ Indian Oil Corporation Ltd.
➢ Coal India Ltd.
➢ Bharat Heavy Electricals Ltd.
➢ Gas Authority of India Ltd.

111. Which of the following is/are the tools likely to be used by the RBI to mop up the extra liquidity in the
system?
1. Market Stabilization Scheme
2. Open market operations
3. Standing Deposit Facility
Select the correct answer using the code given below.
(a) 1 and 2 only (b) 2 only
(c) 3 only (d) 1, 2 and 3
Answer (d)
Explanation
India is planning to introduce a new monetary-policy tool in the financial year 2017-18 to better
manage a banking system swimming in excess cash. The so-called Standing Deposit Facility, or SDF,
will help the Reserve Bank of India absorb surplus funds without having to provide lenders
collateral in exchange. In 2014, a panel led by RBI Governor Urjit Patel -- who was at that time a


deputy -- had proposed the introduction of the SDF as part of measures to improve the monetary-
policy framework.

112. Which of the following is/are Small Saving Schemes of Government of India?
1. Sukanya Samridhi Yojna
2. Kisan Vikas Patra
3. National Savings Certificate
Select the correct answer using the code given below.
(a) 1 only (b) 2 and 3 only
(c) 1 and 3 only (d) 1, 2 and 3
Answer (d)
Explanation
Small saving schemes in India are one of the country’s most viable financial options. Their main
aim is to provide financial help to the economically underprivileged section of the Indian
population, a group that makes up a huge portion of the country’s population. They become
instrumental for generation of money to be spent on development.
The providers of small savings schemes want to achieve this by providing the common people with
investment options that offer high returns without high risk.
The major savings scheme options are
• Sukanya Samridhi Yojna
• Kisan Vikas Patra
• National Savings Certificate
• Public Provident Fund
• Senior Citizens Savings Scheme etc.

113. Which of the following statements is/are correct?


1. Countervailing Duties are imposed in order to counter the negative impact of export subsidies to
protect domestic producers.
2. An anti-dumping duty is a protectionist tariff that a domestic government imposes on foreign
imports that it believes are priced below fair market value.
Select the correct answer using the code given below.
(a) 1 only (b) 2 only
(c) Both 1 and 2 (d) Neither 1 nor 2
Answer (c)
Explanation
An anti-dumping duty is a protectionist tariff that a domestic government imposes on foreign
imports that it believes are priced below fair market value. Dumping is a process where a company
exports a product at a price lower than the price it normally charges on its own home market. To
protect local businesses and markets, many countries impose stiff duties on products they believe are
being dumped in their national market.
In cases foreign producers attempt to subsidize the goods being exported by them so that it
causes domestic production to suffer because of a shift in domestic demand towards cheaper imported
goods, the government makes mandatory the payment of a countervailing duty on the import of such
goods to the domestic economy.

114. Consider the following statements regarding the National Investment and Infrastructure Fund (NIIF):
1. The objective of NIIF would be to maximize economic impact mainly through infrastructure
development in commercially viable projects, both green field and brown field, including stalled
projects.
2. Its governing council will have the participation from the experts of the infrastructure sector.
3. Government will be the majority shareholder of the fund.
Which of the statements given above is/are correct?
(a) 1 and 2 only (b) 2 and 3 only
(c) 3 only (d) 1, 2 and 3
Answer (a)
Explanation
National Investment and Infrastructure Fund (NIIF) is a major initiative by the Government of India
to give fillip to Infrastructure sector and spur economic growth. Its creation was announced in
the Union Budget 2015-16. The objective of NIIF would be to maximize economic impact mainly
through infrastructure development in commercially viable projects, both greenfield and
brownfield, including stalled projects. It could also consider other nationally important projects, for
example, in manufacturing, if commercially viable.


The proposed corpus of NIIF is ₹40,000 Crores (around USD 6 Billion). The initial authorized
corpus of NIIF would be ₹20,000 crore, which may be raised from time to time, as decided by Ministry
of Finance. Government can provide upto ₹20000 crore per annum into these funds. Government's
contribution/share in the corpus will be 49% in each entity set up as an alternate Investment Fund
(AIF) and will neither be increased beyond, nor allowed to fall below, 49%. The whole of 49% would
be contributed by Government directly. Rest is open for contribution from others.
Governing Council of NIIF is an advisory body - chaired by the Finance Minister. It shall comprise
of government representatives, eminent economists, professionals from the Infrastructure Sector and
representatives from the non-government contributors.
Government is in the process of setting up the two sub funds under the NIIF- one in clean energy
fund and other in focus on highways projects.

115. Economic Survey 2016-17 has criticised the global credit rating agencies for India’s lower credit rating
though the Government of India carried out a lot of reforms since 2014. Which of the following factors
is/ are the major hindrances to the India’s credit rating upgrade as per the credit rating agencies?
1. History of credit default
2. Low per capita GDP
3. Combined Government fiscal deficit
Select the correct answer using the code given below.
(a) 2 and 3 only (b) 1 and 2 only
(c) 1 and 3 only (d) 1, 2 and 3
Answer (a)
Explanation
The Economic Survey 2016-17, noted that India’s ratings have remained stuck at the much lower
level of BBB, despite the country’s “dramatic” improvement in growth and macro-economic stability
since 2014. In November, 2016, the Standard & Poor ruled out the scope of a ratings upgrade for
India, mainly on the grounds of its low per capita GDP and relatively high fiscal deficit.
India has no history of default loans

116. Ratan Watal Committee, sometimes seen in the news, is related to


(a) Monetary policy committee
(b) Bitcoin
(c) Digital payments
(d) Small Finance Banks
Answer (c)
Explanation
The Committee on Digital Payments constituted by the Ministry of Finance, Department of
Economic Affairs under the Chairmanship of Shri. Ratan P. Watal, Principal Advisor, NITI Aayog and
former Finance Secretary to the Government of India. In its Report, the Committee has recommended
that a medium term strategy for accelerating growth of Digital Payments in India with a
regulatory regime which is conducive to bridging the Digital divide by promoting competition, open
access & interoperability in payments. The Report recommends inclusion of financially and socially
excluded groups and assimilation of emerging technologies in the market, while safeguarding
security of Digital Transactions and providing level playing to all stakeholders and new players who
will enter this new transaction space. It has suggested inter-operability of the payments system
between banks and non-banks, up-gradation of the digital payment infrastructure and institutions
and a framework to reward innovations and for leading efforts in enabling digital payments.

117. Which of the following scheme/schemes is/are unveiled by RBI for the resolution of the stressed asset
problem?
(a) Sustainable Structuring of Stressed Assets
(b) Asset Quality Review
(c) Private Asset Reconstruction Companies
(d) All of the above
Answer (d)
Explanation
Over the past three years the RBI has implemented a number of schemes to facilitate resolution of the
stressed asset problem.
The 5/25 Refinancing of Infrastructure Scheme:
This scheme offered a larger window for revival of stressed assets in the infrastructure sectors and
eight core industry sectors. Under this scheme lenders were allowed to extend amortisation periods to
25 years with interest rates adjusted every 5 years, so as to match the funding period with the long


gestation and productive life of these projects. The scheme thus aimed to improve the credit profile and
liquidity position of borrowers, while allowing banks to treat these loans as standard in their balance
sheets, reducing provisioning costs. However, with amortisation spread out over a longer period, this
arrangement also meant that the companies faced a higher interest burden, which they found difficult
to repay, forcing banks to extend additional loans (‘ever greening’). This in turn has aggravated the
initial problem.
Private Asset Reconstruction Companies (ARCs):
ARCs were introduced to India under the SARFAESI Act (2002), with the notion that as specialists in
the task of resolving problem loans, they could relieve banks of this burden. However, ARCs have
found it difficult to resolve the assets they have purchased, so they are only willing to purchase loans
at low prices. Asa result, banks have been unwilling to sell them loans on a large scale. Then, in 2014
the fee structure of the ARCs was modified, requiring ARCs to pay a greater proportion of the purchase
price up-front in cash. Since then, sales have slowed to a trickle: only about 5 percent of total NPAs at
book value were sold over 2014-15 and 2015-16.
Strategic Debt Restructuring (SDR):
The RBI came up with the SDR scheme in June 2015 to provide an opportunity to banks to convert
debt of companies (whose stressed assets were restructured but which could not finally fulfil the
conditions attached to such restructuring) to 51 per cent equity and sell them to the highest bidders,
subject to authorization by existing
shareholders. An 18-month period was envisaged for these transactions, during which the loans could
be classified as performing. But as of end-December 2016, only two sales had materialized, in part
because many firm’s remained financially unviable, since only a small portion of their debt had been
converted to equity.
Asset Quality Review (AQR):
Resolution of the problem of bad assets requires sound recognition of such assets. Therefore, the RBI
emphasized AQR, to verify that banks were assessing loans in line with RBI loan classification rules.
Any deviations from such rules were to be rectified by March 2016.
Sustainable Structuring of Stressed Assets (S4A):
Under this arrangement, introduced in June 2016, an independent agency hired by the banks will
decide on how much of the stressed debt of a company is ‘sustainable’. The rest (‘unsustainable’) will
be converted into equity and preference shares. Unlike the SDR arrangement, this involves no change
in the ownership of the company.

118. Marginal Cost of fund based Lending Rate (MCLR) is a new methodology to calculate the lending rate of
the commercial banks. Which of the following elements is/are considered while calculating MCLR?
1. Marginal cost of funds
2. Tenur premium
3. Net Demand and Time Liability of the bank
Select the correct answer using the code given below.
(a) 1, 2 and 3 (b) 2 and 3
(c) 1 and 2 (d) 1 and 3
Answer (c)
Explanation
MCLR is comprised of the following components
1. Marginal cost of funds
2. Negative carry on account of CRR
3. Operating cost
4. Tenor premium

119. Which one of the following statements regarding the shadow banks is not correct?
(a) They are not subjected to regulatory oversight
(b) They carry out traditional banking functions
(c) They were one of the main reasons for the global financial crisis in 2008
(d) None of the above
Answer (b)
Explanation
A shadow banking system refers to the financial intermediaries involved in facilitating the creation of
credit across the global financial system but whose members are not subject to regulatory oversight.
The shadow banking system also refers to unregulated activities by regulated institutions. Examples of
intermediaries not subject to regulation include hedge funds, unlisted derivatives and other unlisted
instruments, while examples of unregulated activities by regulated institutions include credit default
swaps. The shadow banking system has escaped regulation primarily because it does not accept
traditional bank deposits. As a result, many of the institutions and instruments have been able to


employ higher market, credit and liquidity risks, and do not have capital requirements commensurate
with those risks. Subsequent to the subprime meltdown in 2008, the activities of the shadow banking
system came under increasing scrutiny and regulations.

120. With reference to the Soil Health Card (SHC), consider the following statements:
1. Using a grid-wise approach, representative soil samples from the fields are tested for nutrient
content in designated chemical laboratories.
2. Both macro and micro nutrients needed by the soil are identified and translated into specific,
measured quantities of fertilisers required.
Which of the statements given above is/are correct?
(a) 1 only (b) 2 only
(c) Both 1 and 2 (d) Neither 1 nor 2
Answer (c)
Explanation
The Prime Minister launched a nation-wide “Soil Health Card” (SHC) scheme in early 2015 to
rejuvenate India’s exhausted soil. Using a grid-wise approach, representative soil samples from the
fields are tested for nutrient content in designated chemical laboratories. Accordingly, macro and
micro nutrients needed by the soil are identified and translated into specific, measured quantities of
fertilisers required. This information, printed on the SHC, is made available to the farmers in that grid
through the state agricultural departments. Thirty million SHCs were issued in 2015-16 and the
Ministry of Agriculture aims to cover the entire farming population by 2018-19. In addition, on a pilot
basis, the soil health information is made available at fertiliser purchase points —Primary Agricultural
Credit Societies (PACS) and POS devices-enabled fertiliser retail shops.

121. With reference to ‘Gokul Gram’ under National Gokul mission, which of the following statements is/are
correct?
1. It will act as centres for development of Indigenous Breeds.
2. It will be established in the native breeding tracts and near metropolitan cities for housing the
urban cattle.
Select the correct answer using the code given below.
(a) 1 only (b) 2 only
(c) Both 1 and 2 (d) Neither 1 nor 2
Answer (c)
Explanation
The National Gokul Mission was launched for the preservation and promotion of indigenous
breeds of cows under National Programme for Bovine Breeding and Dairy Development for the
first time in the country. Establishment of Integrated Indigenous Cattle Centres viz “Gokul Gram”
is a sub component of it. Under this component it is proposed to establish Integrated Indigenous
Cattle Centres or Gokul Grams in the breeding tracts of indigenous breeds. Gokul Grams will be
established in: i) the native breeding tracts and ii) near metropolitan cities for housing the urban
cattle. Gokul Gram will act as Centres for development of Indigenous Breeds and a dependable
source for supply of high genetic breeding stock to the farmers in the breeding tract. The Gokul
Gram will be self-sustaining and will generate economic resources from sale of milk, organic
manure, vermi-composting, urine distillates, and production of electricity from bio gas for in
house consumption and sale of animal products.

122. With reference to benefits of eNAM, which of the following statements is/are correct?
1. It will reduce the gap between farm to fork prices (the prices farmers get and prices paid by retail
consumers).
2. Bulk buyers of agri commodities will be able to conduct e-procurement operations across regions.
3. It eliminates the number of intermediaries and improves the scope for more income for farmers.
Select the correct answer using the code given below.
(a) 1 only (b) 2 and 3 only
(c) 1 and 2 only (d) 1, 2 and 3
Answer (c)
Explanation
With NAM in place, farmers will have access to many more buyers, including from outside a State.
Thus, a farmer can choose from where and to whom he wants to sell his produce. Similarly,
buyers will have more freedom to choose from where they want to buy. With middlemen mostly out of
the scene, NAM is expected to create a Pan-India common market with freer inter-State
movement of agri commodities. Real-time monitoring of prices and transparency in operations,
such as weighing, pricing, billing and presence of a competitive marketing platform, will lead to better
price discovery. It will reduce the gap between farm to fork prices (the prices farmers get and prices

paid by retail consumers).It reduces the number of intermediaries and improves the scope for more
income for farmers. Bulk buyers of agri commodities such as the Food Corporation of India may be
able to conduct e-procurement operations across regions, at transparent prices.

123. With reference to Minimum Support Price (MSP), consider the following statements:
1. MSP is a form of market intervention by the Government of India to insure all agricultural
producers against any sharp fall in farm prices.
2. MSPs have also been used as a tool by the Government to incentivise farmers to grow crops that
are in short supply.
Which of the statements given above is/are correct?
(a) 1 only (b) 2 only
(c) Both 1 and 2 (d) Neither 1 nor 2
Answer (b)
Explanation
Minimum Support Price (MSP) is a form of market intervention by the Government of India to
insure agricultural producers growing major agriculture commodities (24 commodities) against any
sharp fall in farm prices during bumper production years. The major objectives are to support the
farmers from distress sales and to procure food grains for public distribution. MSPs have also been
used as a tool by the Government to incentivise farmers to grow crops that are in short supply.

124. With reference to Public Distribution System (PDS), consider the following statements:
1. For Public Distribution System, FCI has assumed the responsibility for procurement, storage,
transportation and bulk allocation of food grains to the State Governments.
2. Foodgrains are issued by the FCI to the States at a uniform Central Issue Price (CIP).
Which of the statements given above is/are correct?
(a) 1 only (b) 2 only
(c) Both 1 and 2 (d) Neither 1 nor 2
Answer (c)
Explanation
In simple terms, Public distribution system (PDS) is a government-sponsored chain of shops
entrusted with the work of distributing basic food and non-food commodities to the needy sections
of the society at very cheap prices. The Central Government, through FCI, has assumed the
responsibility for procurement, storage, transportation and bulk allocation of food grains to the
State Governments. On an average, about 15-16 million tonnes of foodgrains are issued by the FCI to
the States at a uniform Central Issue Price (CIP) which is much less than the economic cost
incurred by the Central Government by way of procurement, storage, transport and distribution.

125. A ‘wilful default’ would be deemed to have occurred if


1. the unit has defaulted eventhough it has the capacity to repay the loans
2. the unit has defaulted and has diverted the funds for other purposes.
3. the unit has defaulted and the fund has not utilized for the specific purpose to which it is availed.
4. the unit has defaulted and has sold off the mortgaged asset without knowledge of the lender
Select the correct answer using the codes given below.
(a) 1 only (b) 2 and 3 only
(c) 1 and 3 only (d) 1, 2, 3 and 4
Answer (d)
Explanation
A ‘wilful default’ would be deemed to have occurred if any of the following events is noted:
1. The unit has defaulted in meeting its payment / repayment obligations to the lender even when it
has the capacity to honour the said obligations.
2. The unit has defaulted in meeting its payment / repayment obligations to the lender and has not
utilised the finance from the lender for the specific purposes for which finance was availed of but
has diverted the funds for other purposes.
3. The unit has defaulted in meeting its payment / repayment obligations to the lender and has
siphoned off the funds so that the funds have not been utilised for the specific purpose for which
finance was availed of, nor are the funds available with the unit in the form of other assets.
4. The unit has defaulted in meeting its payment / repayment obligations to the lender and has also
disposed off or removed the movable fixed assets or immovable property given for the purpose of
securing a term loan without the knowledge of the bank / lender.
126. The establishment of ‘Small Finance Banks’ is being allowed in India to promote the financial
inclusion. Which of the following statements is/are correct in this context?


1. Existing Non-Banking Finance Companies (NBFCs), Micro Finance Institutions (MFIs), and Local
Area Banks (LABs) that are owned and controlled by residents can opt for conversion into Small
Finance Banks.
2. Small Finance Banks will be required to extend 50 per cent of its Adjusted Net Bank Credit (ANBC)
to the sectors eligible for classification as priority sector lending (PSL) by the Reserve Bank.
3. Small Finance Banks can undertake both deposit and lending activities.
Select the correct answer using the code given below.
(a) 1 and 2 only (b) 1 and 3 only
(c) 2 only (d) 1, 2 and 3
Answer (b)
Explanation
The objectives of setting up of Small Finance Banks will be to further financial inclusion by (a)
provision of savings vehicles, and (b) supply of credit to small business units; small and marginal
farmers; micro and small industries; and other unorganised sector entities, through high technology-
low cost operations. Resident individuals/professionals with 10 years of experience in banking and
finance; and companies and societies owned and controlled by residents will be eligible to set up small
finance banks. Existing Non-Banking Finance Companies (NBFCs), Micro Finance Institutions
(MFIs), and Local Area Banks (LABs) that are owned and controlled by residents can also opt for
conversion into small finance banks.
The Small Finance Bank shall primarily undertake basic banking activities of acceptance of
deposits and lending to unserved and underserved sections including small business units, small
and marginal farmers, micro and small industries and unorganised sector entities.
The Small Finance Banks will be required to extend 75 per cent of its Adjusted Net Bank Credit
(ANBC) to the sectors eligible for classification as priority sector lending (PSL) by the Reserve Bank.

127. Which of the following do not come under the purview of ‘Place of Effective Management’ (PoEM)
guidelines recently unveiled by Central Board of Direct Taxes?
1. Shell companies that are created to evade taxes through establishing overseas subsidiaries.
2. Firms with an annual turnover of less than ₹ 50 Crore.
3. All the businesses with active business operations overseas.
4. MNC’s with their units in India.
Select the correct answer using the code given below.
(a) 1, 2 and 3 only (b) 1, 2 and 4 only
(c) 2, 3 and 4 only (d) 1, 3 and 4 only
Answer (c)
Explanation
‘Place of Effective Management’ (PoEM) is an internationally recognised test for determination of
residence of a company incorporated in a foreign jurisdiction. Most of the tax treaties entered into by
India recognises the concept of the place of effective management for determination of residence of a
company as a tie-breaker rule for the avoidance of double taxation.
Recently Government of India issued clarifying rules on determining the companies’ place of effective
management (POEM), in order to target the shell companies that are created to evade taxes
through establishing overseas subsidiaries.
However, it excluded from its purview firms with an annual turnover of less than ₹50 Crore and all
the businesses with active business operations overseas.
Thus, effectively, Indian companies with genuine business operations abroad and the MNC’s with
their units in India are not covered under the POEM guidelines.

128. Which of the following States have the ‘special category status’?
1. Assam
2. Meghalaya
3. Sikkim
4. Andhra Pradesh
Select the correct answer using the code given below.
(a) 1, 2, 3 and 4 (b) 2 and 4 only
(c) 1 and 3 only (d) 1, 2 and 3 only

Answer (d)
Explanation
The states having the special category status are Jammu and Kashmir, Uttarakhand, Himachal
Pradesh, Sikkim, Assam, Arunachal Pradesh, Manipur, Nagaland, Tripura, Meghalaya and
Mizoram.


The criteria for granting special status were 1) a lot of hilly terrain 2) economic and social
backwardness and lack of infrastructure 3) a large tribal population 4) international borders 5) non-
viable nature of finances.

129. Which one of the following pairs is correctly matched?


Country Stock indices
(a) Japan : KOSPI
(b) UK : NASDAQ
(c) USA : FTSE
(d) Brazil : Bovespa
Answer (d)
Explanation
Japan : Nikkei
South Korea : KOSPI
UK : FTSE
USA : NASDAQ, Dow Jones

130. What do you mean by ‘footloose’ industry?


(a) An industry that can be established anywhere
(b) Another name of Arsenic industry that cause ‘black foot’ disease
(c) A start up industry having no government support
(d) An industry that has high stressed assets
Answer (a)
Explanation
Footloose industries that is not tied to any particular location or county, and can relocate across
national borders in response to changing economic conditions.

131. Fiscal slippage means


(a) the actual fiscal deficit is more than the expected limit
(b) the actual revenue deficit is more than the expected limit
(c) the decrease in the amount of foreign direct investment compared to previous financial year
(d) the decrease in the amount of foreign portfolio investment compared to previous financial year
Answer (a)
Explanation
If the actual fiscal deficit is more than what was expected it is called as fiscal slippage. For
example, in the budget, estimated fiscal deficit was 3.5% but the actual deficit at the end of the
financial year is 5%, then it is called fiscal slippage.

132. Twin deficit includes


(a) Fiscal deficit and revenue deficit
(b) Fiscal deficit and current account deficit
(c) Current account deficit and effective revenue deficit
(d) Current account deficit and revenue deficit
Answer (b)
Explanation
Economies that have both fiscal deficit and current account deficit are referred as having twin
deficit.

133. Which of the following statements regarding the Scheme for Sustainable Structuring of Stressed Assets
(S4A) is/are correct?
1. The S4A is launched by Reserve Bank of India.
2. It is helpful for the banks manage its Non-Performing Assets.
3. There is no change in the ownership of the company, when a company avails benefit under S4A.
4. The scheme is available for corporates of non-wilful defaulter variety.
Select the correct answer using the code given below.
(a) 1 only (b) 2 and 3 only
(c) 1 and 3 only (d) 1, 2, 3 and 4
Answer (d)
Explanation
The S4A is RBI’s latest attempt to try and chip away at the mountain of stressed loans smothering
Indian banks. The scheme tries to strike a compromise between banks with problem accounts and


corporate defaulters of the non-wilful variety, by converting a portion of large loan accounts
into equity shares.
For a distressed company to be eligible for S4A, the RBI has laid down three conditions.
1. The project must be operating and already generating cash.
2. The total loans to the entity should be ₹500 crore or more.
3. The lending banks are required to hire an independent agency to evaluate how much of the debt
is ‘sustainable’. For the loan to be eligible for S4A, at least 50 percent of it should be
‘sustainable’.
While RBI’s earlier ideas to resolve bad loans such as Strategic Debt Restructuring (SDR) required
banks to oust existing promoters, S4A allows the incumbent management to continue, as long the
default isn’t wilful.
By straightaway allowing as much as half of the loan to be written off through conversion into
equity, S4A gives corporate borrowers the opportunity to trim their debt pile and start afresh.

134. Recently Union Government has changed the investment policy in the aviation sector. Consider the
following statements regarding the same:
1. The amendments allow 100% FDI under automatic route in Brownfield Airport projects.
2. The foreign investment up to 100% was allowed under automatic route in Scheduled Air Transport
Service/ Domestic Scheduled Passenger Airline and regional Air Transport.
Which of the statements given above is/are correct?
(a) 1 only (b) 2 only
(c) Both 1 and 2 (d) Neither 1 nor 2
Answer (a)
Explanation
• The earlier FDI policy on Airports permitted 100% FDI under automatic route in Greenfield
Projects and 74% FDI in Brownfield Projects under automatic route. FDI beyond 74% for
Brownfield Projects is under government route. With a view to aid in modernization of the existing
airports to establish a high standard and help ease the pressure on the existing airports, 100%
FDI under automatic route has now been permitted in Brownfield Airport projects.
• As per the earlier FDI policy, foreign investment up to 49% was allowed under automatic route in
Scheduled Air Transport Service/ Domestic Scheduled Passenger Airline and regional Air
Transport Service. This limit has now been raised to 100%, with FDI up to 49% permitted
under automatic route and FDI beyond 49% through Government approval. For NRIs, 100%
FDI will continue to be allowed under automatic route. Foreign airlines would continue to be
allowed to invest in capital of Indian companies operating scheduled and non-scheduled air-
transport services up to the limit of 49% of their paid up capital.

135. With reference to National Food Security Act (NFSA) 2013, which of the following statement is/are
correct?
1. It aims at covering 50 per cent of rural and 25 per cent of urban population.
2. It promises to provide beneficiaries with rice/wheat/coarse cereals at `3/2/1 Kg through Public
Distribution System (PDS).
Select the correct answer using the code given below.
(a) 1 only (b) 2 only
(c) Both 1 and 2 (d) Neither 1 nor 2
Answer (b)
Explanation
The National Food Security Act (NFSA) 2013, aims at covering overall 67% of population (75% of
rural and 50% of urban population) and promised to give them rice/wheat/coarse cereals at
Rs.3/2/1/Kg through Public Distribution System (PDS).

136. With reference to States joining Centre’s ‘National Agriculture Market or eNAM’, consider the following
statements:
1. The State will have to amend the APMC Act to provide for electronic auctioning.
2. The State APMC Act must provide for issuance of licenses to anyone in India including private
players to trade through NAM in local mandis.
3. There must be a single license for each State that covers all mandis within the State with a single-
point levy of transaction fee.
Which of the statements given above is/are correct?
(a) 1 only (b) 2 and 3 only
(c) 1 and 3 only (d) 1, 2 and 3
Answer (d)
Explanation

The Centre Government has set three basic criteria for a State to join Centre’s NAM. First, the State
will have to amend the APMC Act to provide for electronic auctioning. Secondly, the State APMC Act
must provide for issuance of licenses to anyone in India including private players to trade through
NAM in local mandis. Thirdly, and most importantly, there must be a single license for each State
that covers all mandis within the State with a single-point levy of transaction fee.

137. Which of the following statements regarding the Infrastructure Investment Trusts (InvITs) is/are
correct?
1. InvITs allow one to invest in infrastructure project like roads and power.
2. InvITs are allowed only in the completed projects.
3. IRB Infrastructure Development Ltd recently issued first InvIT in India.
4. They provide an alternate fund option to developers as it provide necessary short term capital to
infrastructure development.
Select the correct answer using the code given below.
(a) 1 only (b) 2 and 3
(c) 1 and 3 (d) 1, 2 and 4
Answer (c)
Explanation
An Infrastructure Investment Trust (InvITs) is like a mutual fund, which enables direct investment
of small amounts of money from possible individual/institutional investors in infrastructure to
earn a small portion of the income as return. InvITs work like mutual funds or real estate investment
trusts (REITs) in features. InvITs can be treated as the modified version of REITs designed to suit the
specific circumstances of the infrastructure sector.
Two types of InvITs are there. One in completed and revenue generation infrastructure projects and
the other which has the flexibility to invest in completed or under construction projects.
Recently IRB Infrastructure Development Ltd issued first InvIT IPO in India.
InvITs are designed to attract low cost, long term capital and the underlying focus is to reduce the
pressure on the banking system as well as generating fresh equity capital in the infrastructure
sector.

138. Which of the following statements regarding the Real Estate Regulatory Authority (RERA) is not
correct?
(a) According to Real Estate (Regulation and Development) Act, 2016, each State and Union territory
will have its own regulator and set of rules to govern the functioning of the regulator
(b) For any delay in completion of project, the developer has to pay the penal interest to the RERA
(c) A consent of two-third of the buyers is mandatory for approval of any deviation from the existing
plan
(d) 70% of the sale proceeds of the project go to a separate account to finance construction
Answer (b)
Explanation
According to RERA, each state and Union territory will have its own regulator and set of rules to
govern the functioning of the regulator. Centre has drafted the rules for Union territories including the
national Capital.
The promoter of a real estate development firm has to maintain a separate escrow account for
each of their projects. A minimum 70 per cent of the money from investors and buyers will have to be
deposited. This money can only be used for the construction of the project and the cost borne
towards the land.
RERA requires builders to submit the original approved plans for their ongoing projects and the
alterations that they made later.
It will be the responsibility of each state regulator to register real estate projects and real estate
agents operating in their state under RERA. The details of all registered projects will be put up on
a website for public access.
The regulator will ensure protection to buyers in this matter for five years from the date of
possession. If any issue is highlighted by buyers in front of the regulator in this period including in
quality of construction and the provision of services, the developer will have to rectify the same in a
matter of 30 days.
Developers can’t invite, advertise, sell, offer, market or book any plot, apartment, house,
building, investment in projects, without first registering it with the regulatory authority.
Furthermore, after registration, all the advertisement inviting investment will have to bear the unique
RERA registration number. The registration no. will be provided project-wise.
If the promoter defaults on delivery within the agreed deadline, they will be required to return the
entire money invested by the buyers along with the pre agreed interest rate mentioned in the
contract based on the model contract given by RERA.


If the buyer chooses not to take the money back, the builder will have to pay monthly interest on
each delay month to the buyer till they get delivery.
To add further security to buyers, RERA mandates that developers can’t ask more than 10 per
cent of the property’s cost as an advanced payment booking amount before actually signing a
registered sale agreement.
The regulator will have the power to fine and imprison errant builders based on a case by case
basis. The imprisonment can go up to a period of three years for a project.

139. Which one of the following is a purpose of ‘Atal Mission for Rejuvenation and Urban Transformation’, a
scheme of the Government?
(a) It aims to transform slum areas by building homes for slum dwellers
(b) It aims to develop and upgrade living conditions and infrastructure in selected 100 cities across the
country
(c) It is aimed at preserving and revitalizing the soul and unique character of the heritage cities in
India
(d) It is aimed at transforming 500 cities and towns into efficient urban living spaces over a period of
five years
Answer (d)
Explanation
Atal Mission for Rejuvenation and Urban Transformation (AMRUT) was launched on 24th June,
2015. AMRUT along with smart cities were jointly planned and launched by the government to
transform urban living conditions through infrastructure upgradation. AMRUT is aimed at
transforming 500 cities and towns into efficient urban living spaces over a period of five years. The
purpose of Atal Mission for Rejuvenation and Urban Transformation (AMRUT) is to (i) ensure that
every household has access to a tap with assured supply of water and a sewerage connection; (ii)
increase the amenity value of cities by developing greenery and well maintained open spaces
(parks); and (iii) reduce pollution by switching to public transport or constructing facilities for non-
motorized transport (e.g. walking and cycling).

140. What is/ are the purpose/purposes of Government’s ‘Lucky Grahak Yojana and ‘Digi-Dhan Vyapar
Yojana’?
1. To promote digital payment methods to encourage consumers and merchants
2. To give cash awards to consumers and merchants who utilize digital payment instruments for
personal consumption expenditures.
3. To incentivize digital transactions so that electronic payments are adopted by all sections of the
society, especially the poor and the middle class.
Select the correct answer using the code given below.
(a) 1 only (b) 2 and 3 only
(c) 1 and 3 only (d) 1, 2 and 3
Answer (d)
Explanation
The Government of India seeks to promote digital payment methods to encourage consumers and
merchants to increasingly shift to these payment modes. The two schemes, namely Lucky Grahak
Yojana for Consumers and Digi-Dhan Vyapar Yojana for Merchants, seek to nudge people towards
significantly higher usage of digital transactions through the offer of incentives. The Lucky Grahak
Yojana and the Digi-Dhan Vyapar Yojana offers cash awards to consumers and merchants who utilize
digital payment instruments for personal consumption expenditures. The scheme specially focuses
on bringing the poor, lower middle class and small businesses into the digital payment fold. The
National Payment Corporation of India (NPCI) is the implementing agency for this scheme. The
scheme is operational during the period 25th December, 2016 to 14th April 2017.

141. The Inclusive Growth and Development Report is published by


(a) World Economic Forum
(b) UN Development Programme
(c) International Monetary Fund
(d) World Bank
Answer (a)
Explanation
India has been ranked 60th among 79 developing economies, below neighbouring China and
Pakistan, in the inclusive development index, according to a WEF report. WEF’s ‘Inclusive Growth
and Development Report 2017’ said that most countries are missing important opportunities to raise
economic growth and reduce inequality at the same time because the growth model and measurement
tools that have guided policymakers for decades require significant readjustment.


The other major reports of WEF are
• Travel and Tourism Competitiveness report
• Global Energy Architecture report
• Global Risks Report
• Global Enabling Trade Report
• Global Gender Gap report
• Global Competitiveness Report
• Global Information Technology Report

142. Which of the following statement regarding ‘data exclusivity’ sometimes seen in the news, is not
correct?
(a) It prevents drug regulators from referring to or relying on data submitted by an originator company
relating to a drug’s safety and efficacy while approving bioequivalent versions of the same drug
(b) The Agreement on Trade-Related Aspects of Intellectual Property Rights (TRIPS) does not mandate
data exclusivity
(c) Data exclusivity would prevent market entry of generic versions of the drug
(d) India is the only country that so far adopted the data exclusivity provisions as part of its
intellectual property policy
Answer (d)
Explanation
Data exclusivity is a kind of intellectual property protection wherein clinical trial and other data
submitted by an originator drug company cannot be used or relied upon by a drug regulatory
authority to approve a generic version of that drug for a certain period of time. The notion is that
without such protection, the originator company lacks the necessary commercial incentive to conduct
expensive trials and take a potential drug to the market. Blocking generic entry for some years will,
the theory goes, help drug companies invest in clinical trials.
India has long resisted U.S. and EU pressure to institute data exclusivity norms, seeing it as a
barrier to generic entry and more affordable drug prices.
The Agreement on Trade-Related Aspects of Intellectual Property Rights (TRIPS) does not mandate
data exclusivity. Providing data exclusivity is a TRIPS-plus measure.

143. Which one of the following statement regarding cess is not correct?
(a) It is levied only by Union government
(b) It is levied for specific purposes.
(c) It is a tax on tax.
(d) It is levied on goods as well as services
Answer (a)
Explanation
Cess is a tax additionally levied as a percentage of existing tax amount for a specific purpose.
Hence it is a tax on tax. For example, in budget 2016-17 Union government introduced Krishi Kalyan
cess of 0.5% over the service tax and the amount collected used solely towards financing activities
for the improvement of agriculture and farmer welfare. In the same budget the government doubled
the clean environment cess on the coal, lignite and peat. So it is levied on both goods as well as
services.
Both Union as well as state governments use this way to raise extra money to finance its activities.

144. With reference to the sovereign credit rating, consider the following statements:
1. Sovereign Credit rating is determined by World Bank.
2. India’s sovereign credit rating is just above junk bond status (BBB-).
3. Sovereign credit rating helps the domestic companies to get the overseas funds at the competitive
rates.
Which of the statements given above is/are correct?
(a) 1 and 2 only (b) 2 and 3 only
(c) 3 only (d) 1 and 3 only
Answer (b)
Explanation
Sovereign Credit ratings are determined by the international credit rating agencies like Moody’s,
Fitch, Standard and Poors etc.
The credit rating of India has remained unchanged for last several years at BBB-, which is the
lowest investment grade, only a grade above the junk status for the government/sovereign bonds.


The government's 'Economic Survey 2017' too has slammed the agencies for following "inconsistent"
standards while rating India vis-a-vis China, saying they have not taken into account reform measures
like GST, which is a "poor" reflection on their credibility.
Fitch had last upgraded India's sovereign rating from BB+ to BBB- with stable outlook on August 1,
2006. Later, it changed the outlook to negative in 2012 and then again to stable in the following year,
though it kept the rating unchanged at the lowest investment grade.
Ratings are considered important as they help domestic companies raise overseas funds at
competitive rate.

145. With reference to ‘Regional Connectivity Scheme (UDAN)’, consider the following statements:
1. It is aimed at facilitating / stimulating regional air connectivity by making it affordable.
2. It is only intended to revive under-served airports of the country.
3. Under this scheme, the entire seats available in the flight will be capped at Rs. 2,500 per hour’s
flight.
Which of the statements given above is/are correct?
(a) 1 only (b) 2 and 3 only
(c) 1 and 3 only (d) 1, 2 and 3

Answer (a)
Explanation
Regional Connectivity Scheme (UDAN - Ude Desh ka Aam Naagrik) is an innovative scheme to
develop the regional aviation market. The Ministry of Civil Aviation (MoCA) launched the scheme on
21st October, 2016. Primary objective of RCS is to facilitate / stimulate regional air connectivity
by making it affordable. It also intends to revive un-served and under-served airports, promote
tourism, increase employment and promote balanced regional growth. Under this scheme, half of
the seats on the plane will be capped at Rs. 2,500 per hour’s flight. Government will subsidise the
losses incurred by airlines flying to dormant airports.

146. Union Government of India has recently proposed a new Agricultural Produce and Livestock Marketing
(Promotion and Facilitating) model law to replace the Agricultural Produce Marketing Committee Act,
2003. Which of the following is not the feature of the same?
(a) It allows the farmer to sell his produce to any mandi in the country.
(b) It also proposes private wholesale markets.
(c) It delists fruit and vegetables from the APMC Act.
(d) It has provisions to declare warehouses and cold storages as market sub-yards.
Answer (a)
Explanation
The Centre has reportedly recommended a new model law to replace the restrictive Agricultural
Produce Market Committee (APMC) Act that makes it mandatory for farmers to sell only to
designated middlemen, who then control trade in farm produce and corner the benefit of higher
prices.
The new model law suggests single-point levy with a market fee and single registration or licence
for trade in more than one market and delisting fruit and vegetables from the APMC Act. Intra-
State barriers on the movement of food are proposed to be reduced by allowing farmers to sell not just
in designated mandis (as at present) but to any mandi in the State. The liberalised model law also
proposes private wholesale markets, direct sale by farmers to bulk buyers and promotion of
electronic trading. It also proposes to cap mandi taxes at 1% for food grain and 2% for vegetables,
and pegs the commission agents’ levy at 2% of the total transaction cost. Rightly, the model law also
has provisions to declare warehouses and cold storages as market sub-yards to provide better
access and linkages to farmers.

147. ‘Unicorn club’ sometimes seen in the news is related to


(a) Start ups
(b) Major cotton production countries
(c) Turkish referendum
(d) Belt and Road initiative (BRI)
Answer (a)
Explanation
A unicorn, in the world of business, is a company, usually a start-up that does not have an
established performance record, with a stock market valuation or estimated valuation of more than $1
billion.

148. Which of the following statements with regard to Pradhan Mantri Yuva Yojana is/are correct?
1. This is a scheme on entrepreneurship education and training.


2. The scheme spans over a period of three years (2016-17 to 2018-19) with a project cost of around
Rs. 500 crore.
Select the correct answer using the code given below.
(a) 1 only (b) 2 only
(c) Both 1 and 2 (d) Neither 1 nor 2
Answer (a)
Explanation
Pradhan Mantri YUVA Yojana was launched on 9th November, 2016. This is a scheme on
entrepreneurship education and training. This initiative is to scale up entrepreneurship in the
country and has national and international best practices of learning in entrepreneurship
education. It aims to cover 7 lakh students in 5 years through 3050 institutes. The schemes span
over five years (2016-17 to 2020-21) with a project cost of around Rs. 500 crore. It will also include
easy access to information and mentor network, credit, incubator and accelerator and advocacy
to create a pathway for the youth.

149. Which of the following statements are correct about ‘Saakshar Bharat’ scheme?
1. It is a centrally sponsored scheme which was launched during the Eleventh Five Year Plan.
2. The scheme is to impart functional literacy and numeracy to non-literate and non-numerate adults
3. The scheme applied to women in particular and disadvantaged group in general.
4. The scheme applies to persons above the age of 10 years.
Select the correct answer using the code given below.
(a) 1, 2 and 3 (b) 2, 3 and 4
(c) 1, 2 and 4 (d) 1, 3 and 4
Answer (a)
Explanation
Saakshar Bharat has been devised as the new variant of National Literacy Mission. It is a centrally
sponsored scheme which was launched during the Eleventh Five Year Plan (September, 2009). The
scheme is to impart functional literacy and numeracy to non-literate and non-numerate adults.
Saakshar Bharat will cover all adults in the age group of 15 and beyond though its primary focus
will be on women.

150. Consider the following statements regarding the liquid assets:


1. Currency has high liquidity compared to the savings and demand deposits.
2. Time deposits have low liquidity compared to the savings and demand deposits.
Which of the statements given above is/are correct?
(a) 1 only (b) 2 only
(c) Both 1 and 2 (d) Neither 1 nor 2
Answer (c)
Explanation
The liquid assets in the decreasing order of their liquidity is
Currency>Demand deposits>Savings deposits> Time deposits

151. A member’s quota in IMF determines which of the following?


1. Voting power of the country
2. Amount of fund that a country obliged to provide to IMF.
3. Access to financing
Select the correct answer using the code given below.
(a) 1 only (b) 2 and 3 only
(c) 1 and 3 only (d) 1, 2 and 3
Answer (d)
Explanation
A member's quota determines that country’s financial and organizational relationship with the IMF,
including:
Subscriptions: A member's quota subscription determines the maximum amount of financial
resources the member is obliged to provide to the IMF. A member must pay its subscription in full
upon joining the IMF: up to 25 per cent must be paid in SDRs or widely accepted currencies (such as
the US dollar, the euro, the Japanese yen, or the British pound sterling), while the rest is paid in the
member's own currency.
Voting power: The quota largely determines a member's voting power in IMF decisions. Each IMF
member’s votes are comprised of basic votes plus one additional vote for each SDR 100, 000 of quota.
The 2008 reforms fixed the number of basic votes at 5.502 per cent of total votes. The current share of


basic votes in total votes represents close to a tripling of their share prior to the implementation of the
2008 reforms.
Access to financing: The amount of financing a member can obtain from the IMF (its access limit) is
based on its quota. For example, under Stand-By and Extended Arrangements, a member can borrow
up to 145 per cent of its quota annually and 435 per cent cumulatively. However, access may be
higher in exceptional circumstances.

152. With reference to ‘Pradhan Mantri Krishi Sinchai Yojana’, consider the following statements:
1. It is to be implemented in an area development approach, adopting decentralized state level
planning.
2. This scheme is aimed to achieve convergence of investments in irrigation at the field level.
Which of the statements given above is/are correct?
(a) 1 only (b) 2 only
(c) Both 1 and 2 (d) Neither 1 nor 2
Answer (c)
Explanation
To boost irrigation facilities, Pradhan Mantri Krishi Sinchai Yojana (PMKSY) has been formulated to
provide end-to-end solutions in irrigation supply chain, viz. water sources, and distribution
network and farm level applications.The PMSKY programme will focus on: i) ensuring access to
water for every agriculture farm (har khet Ko Pani); ii) increasing agricultural production and
productivity by increasing availability and efficient use of water; iii) providing flexibility and
autonomy to state in the progress of planning and executing programmes; and iv) ensuring a
holistic approach by way of preparation of comprehensive district and state irrigation plans.
Krishi Sinchayee Yojana with an outlay of Rs. 50,000 crores for a period of 5 years (2015-16 to 2019-
20) to achieve convergence of investments in irrigation at the field level. PMKSY is to be
implemented in an area development approach, adopting decentralized state level planning and
projected execution, allowing the states to draw their irrigation development plans based on
district/blocks plans with a horizon of 5 to 7 years. States can take up projects based on the
District/State Irrigation Plan.

153. Consider the following statements regarding the MSCI Emerging Market Index.
1. It is designed to measure equity market performance in global emerging markets.
2. It consists of indices of all G20 nations.
Which of the statements given above is/are correct?
(a) 1 only (b) 2 only
(c) Both 1 and 2 (d) Neither 1 nor 2
Answer (a)
Explanation
The MSCI Emerging Markets Index is an index created by Morgan Stanley Capital International
(MSCI) designed to measure equity market performance in global emerging markets. It is a float-
adjusted market capitalization index that consists of indices in 23 emerging economies: Brazil,
Chile, China, Colombia, Czech Republic, Egypt, Greece, Hungary, India, Indonesia, Korea, Malaysia,
Mexico, Peru, Philippines, Poland, Qatar, Russia, South Africa, Taiwan, Thailand, Turkey and the
United Arab Emirates.

154. Recently Government of India allowed Foreign Investment in the Food processing sector. Consider the
following sector regarding the same.
1. Government has allowed 100% FDI for trading including through e-commerce, in respect of food
products manufactured or produced in India.
2. 100% FDI is already permitted in manufacturing of food products through automatic route.
Which of the statements given above is/are correct?
(a) 1 only (b) 2 only
(c) Both 1 and 2 (d) Neither 1 nor 2
Answer (c)
Explanation
Government has allowed 100% FDI for trading including through e-commerce, in respect of food
products manufactured or produced in India. 100% FDI is already permitted in manufacturing of
food products through automatic route. This will provide impetus to the foreign investment in food
processing sector, benefit farmers immensely and will create vast employment opportunities.

155. Capital gains tax does not apply to


1. Sovereign Gold Bonds
2. Stocks held in hand for less than 12 months


3. Agricultural land
Select the correct answer using the code given below.
(a) 1 and 2 only (b) 2 only
(c) 1 and 3 only (d) 1, 2 and 3
Answer (c)
Explanation
Any profit from the sale of a capital asset is deemed as ‘capital gains’. Short term Capital Gain tax is
15% for stocks (less than 12 months) and Long term Capital Gains tax for stocks (more than 12
months) is zero. Sovereign Gold Bonds and Agricultural Land is exempted.

156. Human Capital Index is published by


(a) World Economic Forum
(b) UNDP
(c) Sustainable Development Solution Network
(d) Commonwealth
Answer (a)
Explanation
India was ranked low at 105th position globally on a worldwide Human Capital Index, which
measures countries’ ability to nurture, develop and deploy talent for economic growth and was
topped by Finland.
India ranks much below China’s 71st position while Bangladesh, Bhutan and Sri Lanka are also
placed higher on the index released by Geneva-based World Economic Forum (WEF) at its Annual
Meeting of New Champions — also known as ‘Summer Davos’ summit.

157. Which of the following statements regarding the strategic debt restructuring (SDR) is not correct?
1. The scheme is initiated by the Ministry of Finance to deal with the problem of Non-Performing
Assets of Indian Banks.
2. It allows the banks to convert their debt into majority equity holding in the default company.
3. There is no change in the management of the company.
Select the correct answer using the code given below.
(a) 1 and 2 only (b) 2 only
(c) 1 and 3 only (d) 1, 2 and 3
Answer (c)
Explanation
The RBI came up with the SDR scheme in June 2015 to provide an opportunity to banks to convert
debt of companies (whose stressed assets were restructured but which could not finally fulfil the
conditions attached to such restructuring) to 51 percent equity and sell them to the highest bidders,
subject to authorization by existing shareholders. An 18-month period was envisaged for these
transactions, during which the loans could be classified as performing. But as of end-December 2016,
only two sales had materialized, in part because many firms remained financially unviable, since only
a small portion of their debt had been converted to equity.

158. Consider the following statements regarding the Bank Boards Bureau (BBB):
1. BBB help Banks in developing strategies and capital raising plans.
2. BBB appoint the heads of public sector banks and other financial institutions.
3. All the Members and Chairman are appointmented as part time members.
Select the correct answer using the code given below.
(a) 1 only (b) 2 and 3 only
(c) 1 and 3 only (d) 1, 2 and 3
Answer (c)
Explanation
With a view to improve the governance of Public Sector Banks (PSBs), the Government of India has
set up an autonomous Banks Board Bureau (BBB). The Bureau will recommend for selection of
heads - Public Sector Banks and Financial Institutions (even from the private sector, if need be) and
help banks in developing strategies and capital raising plans. It will also advise banks how to go
ahead with mergers and acquisitions. Banks Board Bureau has three ex-officio members and
three expert members in addition to Chairman. All the Members and Chairman will be part time.
The appointments have been made for a period of two years.
BBB, which came to life in 2016, as part of the seven-point Indradhanush plan will constantly
engage with the boards of all public sector banks to formulate appropriate strategies for their growth
and development. It would also be responsible for selection of nonexecutive chairman and non-
official directors on the boards. Presently BBB is chaired by the former CAG Vinod Rai.


159. The Reserve Bank of India (RBI) in May 2016 has issued draft guidelines for the new ‘on-tap’ bank
license policy. Consider the following statements regarding the same:
1. Under this policy, RBI would accept applications and licenses for banks throughout the year.
2. These norms are for private sector only.
3. Large industrial houses are excluded as eligible entities but are permitted to invest in the banks up
to 10 per cent.
4. 100% FDI is allowed in these banks under the automatic route.
Which of the statements given above is/are correct?
(a) 1, 2 and 3 (b) 4 only
(c) 1, 3 and 4 only (d) 1, 2, 3 and 4
Answer (a)
Explanation
The Reserve Bank of India (RBI) in May second week has issued draft guidelines for the new ‘on-tap’
bank license policy.
An 'on-tap' facility would mean the RBI will accept applications and grant license for banks
throughout the year. The policy allows aspirants to apply for universal bank license at any time,
subject to the fulfilment of the set conditions. It is for the first time in post nationalization phase
that there comes such an open bank license policy.
The Reserve Bank of India released “Guidelines for ‘on tap’ Licensing of Universal Banks in the
Private Sector” in May 2016.
Some of the key aspects of the Guidelines include:
(i) resident individuals and professionals having 10 years of experience in banking and finance
at a senior level are also eligible to promote universal banks;
(ii) large industrial houses are excluded as eligible entities but are permitted to invest in the
banks up to 10 per cent;
(iii) The initial minimum paid-up voting equity capital for a bank shall be ₹ five billion. Thereafter,
the bank shall have a minimum net worth of ₹ five billion at all times.
(iv) The foreign shareholding in the bank would be as per the existing foreign direct investment (FDI)
policy subject to the minimum promoter shareholding requirement. At present, the aggregate
foreign investment limit is 74 per cent.
(v) Non-Operative Financial Holding Company (NOFHC) has been made non-mandatory in case of
promoters being individuals or standalone promoting/converting entities who/which do not
have other group entities;
(vi) Not less than 51 percent of the total paid-up equity capital of the NOFHC shall be owned by the
promoter/promoter group, instead being wholly owned by the promoter group; and
(vii) Existing specialised activities have been permitted to be continued from a separate entity
proposed to be held under the NOFHC subject to prior approval from the Reserve Bank and
subject to it being ensured that similar activities are not conducted through the bank as well.

160. Which of the following statements regarding Unified Payment Interface (UPI) is correct?
1. UPI is an instant payment system developed by the National Payments Corporation of India (NPCI),
a RBI regulated entity.
2. UPI is built over the IMPS infrastructure and allows you to instantly transfer money between any
two parties' bank accounts.
3. Bharat Interface for Money is an app that lets you make easy and quick payment transactions
using UPI.
Select the correct answer using the code given below.
(a) 1 only (b) 2 and 3 only
(c) 1 and 3 only (d) 1, 2 and 3
Answer (d)
Explanation
Bharat Interface for Money is an app that lets you make easy and quick payment transactions using
UPI. Developed by the National Payments Corporation of India (NPCI), BHIM is an aggregator for all
UPI-based services offered by banks.
UNIFIED PAYMENTS INTERFACE (UPI)
• An instant payment system developed by the National Payments Corporation of India (NPCI), a
RBI regulated entity.
• UPI is built over the IMPS infrastructure and allows you to instantly transfer money between
any two parties' bank accounts.
• UPI-PIN (UPI Personal Identification Number) is a 4-6 digit secret code the user create/set during
first time registration with any UPI app.
• UPI-PIN is used to authorize all bank transactions using UPI.


161. Which of the following statements/ statements is/are correct about the Securitisation and
Reconstruction of Financial Assets and Enforcement of Security Interest Act (SARFAESI Act)?
1. It empowers the RBI to reduce Non-Performing Assets in the banking system.
2. It is applicable only to the assets which are secured or mortgaged.
3. Agricultural land is exempted from the SARFAESI Act.
Select the correct answer using the code given below.
(a) 1 only (b) 2 and 3 only
(c) 1 and 3 only (d) 1, 2 and 3
Answer (b)
Explanation
SARFAESI ACT
The Securitisation and Reconstruction of Financial Assets and Enforcement of Security Interest Act,
2002 (SARFAESI) empowers Banks / Financial Institutions to recover their non-performing assets
without the intervention of the Court. The Act provides three alternative methods for recovery of non-
performing assets, namely: -
• Securitisation
• Asset Reconstruction
• Enforcement of Security without the intervention of the Court
The provisions of this Act are applicable only for NPA loans with outstanding above Rs. 1.00 lac. NPA
loan accounts where the amount is less than 20% of the principal and interest are not eligible to be
dealt with under this Act.
Non-performing assets should be backed by securities charged to the Bank by way of hypothecation or
mortgage or assignment. Security Interest by way of Lien, pledge, hire purchase and lease not liable for
attachment under sec.60 of CPC, are not covered under this Act
The Act empowers the Bank:
• To issue demand notice to the defaulting borrower and guarantor, calling upon them to
discharge their dues in full within 60 days from the date of the notice.
• To give notice to any person who has acquired any of the secured assets from the borrower to
surrender the same to the Bank.
• To ask any debtor of the borrower to pay any sum due or becoming due to the borrower.
• Any Security Interest created over Agricultural Land cannot be proceeded with.
If on receipt of demand notice, the borrower makes any representation or raises any objection,
Authorised Officer shall consider such representation or objection carefully and if he comes to the
conclusion that such representation or objection is not acceptable or tenable, he shall communicate
the reasons for non acceptance WITHIN ONE WEEK of receipt of such representation or objection.
A borrower / guarantor aggrieved by the action of the Bank can file an appeal with DRT and then with
DRAT, but not with any civil court. The borrower / guarantor has to deposit 50% of the dues before an
appeal with DRAT.
If the borrower fails to comply with the notice, the Bank may take recourse to one or more of the
following measures:
• Take possession of the security
• Sale or lease or assign the right over the security
• Manage the same or appoint any person to manage the same

162. Consider the following statements regarding the National Intellectual Property Rights Policy unveiled
by the Government of India recently:
1. The policy is entirely compliant with the TRIPS plus provisions.
2. It suggests making the Department of Industrial Policy and Promotion (DIPP) the nodal agency for
all IPR issues.
3. The new policy kept the anti-ever greening patent provision i.e. section 3(d) of the Patents Act,
1970 intact.
Which of the statements given above is/are correct?
(a) 1 only (b) 2 and 3 only
(c) 1 and 3 only (d) 1, 2 and 3
Answer (b)
Explanation
India’s National Intellectual Property Rights (IPR) Policy recently. The Policy which is in compliance
with WTO's (World Trade Organisation) agreement on TRIPS (Trade Related aspects of IPRs), aims to
sustain entrepreneurship and boost Prime pet scheme 'Make in India.' Here are the highlights:
>> The Policy aims to push IPRs as a marketable financial asset, promote innovation and
entrepreneurship, while protecting public interest.
>>The plan will be reviewed every five years in consultation with stakeholders.


>> In order to have strong and effective IPR laws, steps would be taken — including review of existing
IP laws — to update and improve them or to remove anomalies and inconsistencies.
>>The policy is entirely compliant with the WTO’s agreement on TRIPS.
>>Special thrust on awareness generation and effective enforcement of IPRs, besides
encouragement of IP commercialisation through various incentives.
>> India will engage constructively in the negotiation of international treaties and agreements in
consultation with stakeholders. The government will examine accession to some multilateral treaties
which are in India's interest, and become a signatory to those treaties which India has de facto
implemented to enable it to participate in their decision making process.
>> It suggests making the department of industrial policy and promotion (DIPP) the nodal agency
for all IPR issues. Copyrights related issues will also come under DIPP’s ambit from that of the
Human Resource Development (HRD) Ministry.
>> Trademark offices have been modernised, and the aim is to reduce the time taken for
examination and registration to just 1 month by 2017. The government has already hired around 100
new examiners for trademarks. Examination time for trademarks has been reduced from 13 months
to 8 months, with the new target being to bring the time down to one month by March 2017.
>> Films, music, industrial drawings will be all covered by copyright.
>> The Policy also seeks to facilitate domestic IPR filings, for the entire value chain from IPR
generation to commercialisation. It aims to promote research and development through tax
benefits.
>> Proposal to create an effective loan guarantee scheme to encourage start-ups.
>> It also says “India will continue to utilise the legislative space and flexibilities available in
international treaties and the TRIPS Agreement.” These flexibilities include the sovereign right of
countries to use provisions such as Section 3(d) and Compulsory Licences for ensuring the
availability of essential and life-saving drugs at affordable prices.
>> The policy left the country’s patent laws intact and specifically did not open up Section 3(d) of
the Patents Act, which sets the standard for what is considered an invention in India, for
reinterpretation.
>> The IPR policy favoured the government considering financial support for a limited period on sale
and export of products based on IPRs generated from public-funded research.

163. Which of the following statements regarding the Foreign Currency Non Resident (FCNR) account is/are
correct?
1. FCNR account is a fixed deposit account.
2. Banks are allowed to maintain the FCNR deposits in any permitted currency.
3. NRIs can own the FCNR accounts jointly with other NRIs or with residents who are close relatives.
Select the correct answer using the code given below.
(a) 1 only (b) 2 and 3 only
(c) 1 and 3 only (d) 1, 2 and 3
Answer (d)
Explanation
An FCNR account is a term deposit account that can be maintained by NRIs and PIOs in foreign
currency. Thus, FCNRs are not savings accounts but fixed deposit accounts.
Prior to 2011, FCNR deposits were allowed to be maintained in six currencies: US dollar, Pound
Sterling (GBP), Euro, Japanese Yen, Australian dollar and Canadian dollar. However, in October 2011,
the RBI decided that authorised dealer banks in India may be permitted to accept FCNR deposits
in any permitted currency. 'Permitted currency' for this purpose would mean a foreign currency
which is freely convertible and popularly include Danish Krone, Swiss Frank and Swedish Krona
among others.
RBI, in 2011, permitted NRIs to hold FCNR accounts jointly with other NRIs or with residents who
are close relatives. In this case, the resident relative can operate the account as a power of attorney
holder.

164. Which of the following statements regarding the Asset Reconstruction Companies is/are correct?
1. ARCs are formed under the SARFAESI act, 2002.
2. ARCs have become one of the best solutions to the Non-Performing Asset problem present in the
banking industry.
Select the correct answer using the code given below.
(a) 1 only (b) 2 only
(c) Both 1 and 2 (d) Neither 1 nor 2
Answer (a)
Explanation


ARCs were introduced to India under the SARFAESI Act (2002), with the notion that as specialists in
the task of resolving problem loans, they could relieve banks of this burden. However, ARCs have
found it difficult to resolve the assets they have purchased, so they are only willing to purchase loans
at low prices. As a result, banks have been unwilling to sell them loans on a large scale. Then, in 2014
the fee structure of the ARCs was modified, requiring ARCs to pay a greater proportion of the
purchase price up-front in cash. Since then, sales have slowed to a trickle: only about 5 percent of
total NPAs at book value were sold over 2014-15 and 2015-16.

165. Which of the following statements regarding the Foreign Direct Investment (FDI) in Single Brand Retail
is not correct?
(a) 100 % FDI is allowed in the Single Brand Retail
(b) Up to 49%, FDI is allowed via automatic route.
(c) Beyond 49%, the foreign investor has to comply with the domestic sourcing norms.
(d) There is a 3 year waiver from domestic sourcing norms for companies having state of the art and
cutting edge technology.
Answer (c)
Explanation
Under the FDI in single brand retailing norms, beyond 51%, the foreign investor has to comply with
domestic sourcing norms i.e. in respect of proposals involving foreign investment beyond 51%,
sourcing of 30% of the value of goods purchased, will be done from India, preferably from MSMEs,
village and cottage industries, artisans and craftsmen, in all sectors.
Government may relax sourcing norms for entities undertaking single brand retail trading of products
having ‘state-of-art’ and ‘cutting-edge’ technology and where local sourcing is not possible. But is upto
3 years only.

166. Section 52(1) (i) of the Copyright act of India provides the exception for the reproduction of the work
without breaching the copyright. Which of the following comes under it?
1. allowing teacher or pupil in the course of instruction
2. as part of the question to be answered in the exam
3. as part of the answer to be given in the exam
Select the correct answer using the code given below.
(a) 1 and 2 only (b) 2 only
(c) 1 and 3 only (d) 1, 2 and 3
Answer (d)
Explanation
In its much awaited judgment in the Delhi University photocopying case (The Chancellor Masters and
Scholars of the University of Oxford v. Rameshwari Photocopy Services), the Delhi High Court has
dismissed the copyright infringement petition initiated in August 2012 by three publishers (Oxford,
Cambridge and Taylor & Francis) against a photocopy shop located in the premises of Delhi University.
This case, which was being closely tracked by students, teachers and the publishing industry alike,
was seen as one with immense significance for questions of access to knowledge.
While the publishers made the argument that the creation of course packs and the photocopying of
academic material for the same amounted to an infringement of the exclusive copyright of the authors
and publishers, the defendants argued that the reproduction of materials for educational purposes fell
within the exceptions to copyright under Section 52(1)(i) of the Copyright Act.
Section 52(1)(i) allows for the reproduction of any work i) by a teacher or a pupil in the course of
instruction; or ii) as part of the questions to be answered in an examination; or iii) in answers to such
questions.

167. Consider the following statements regarding the Krishi Kalyan Surcharge is/are correct?
1. These are levied at the rate of 0.5% on the service tax.
2. The revenue obtained under it goes to the Public Account of India.
3. The revenue is used for the welfare of the agriculture and rural economy.
Select the correct answer using the code given below.
(a) 1 and 2 only (b) 2 and 3 only
(c) 3 only (d) 1, 2 and 3
Answer (c)
Explanation
In order to provide a stable and predictable taxation regime and reduce black money, it was
announced in the budget 2016-17 that domestic tax payers can declare undisclosed income or such
income represented in the form of any asset by paying tax at 30%, and surcharge (means an addition


to the existing tax) at 7.5% and penalty at 7.5%, which is a total of 45% of the undisclosed income.
Such declarants will have immunity from prosecution. The Finance Minister while declaring the above
provision mentioned that Surcharge levied at 7.5% of the undisclosed income will be called as Krishi
Kalyan Surcharge, to be used for agriculture and rural economy.
After ushering in the 0.5 percent Swacch Bharat Cess in 2015, the finance minister proposed the
Krishi Kalyan Cess (KKC) in the February 2016 Budget. With effect from June 1, KKC adds on
another 0.5 percent to service tax. After this latest addition, the service tax rate on all the services
we use stands at 15 percent.
Both cess and surcharge are credited to the Consolidated Fund of India.

168. Which one of the following is not the target of the 12th Five year plan?
(a) Real GDP growth rate of 10 percent
(b) Agricultural Growth rate of 4 percent
(c) Manufacturing growth rate of 10 percent
(d) Increase in green cover by 1 million hectare every year during the plan period
Answer (a)
Explanation
12th Five Year Plan 2012-17 Targets
12th Five Year Plan 2012-17 as per the draft document released by the Planning Commission aims at
a growth rate of 8%. This is the revised rate when compared to the initial approach paper. Other
targets of the Twelfth Five Year Plan in different sectors are listed below.
Vision of 12th Five Year Plan (2012-17)
Twelfth Five Year Plan focuses on Growth – Growth which is
• Faster
• Inclusive
• Sustainable
25 Core monitorable Targets of the 12th Five Year Plan (2012-17)
Economic Growth
• Real GDP growth at 8%.
• Agriculture growth at 4%.
• Manufacturing growth at 10%.
• Every state must attain higher growth rate than the rate achieved during 11th plan.
Poverty and Employment
• Poverty rate to be reduced by 10% than the rate at the end of 11th plan.
• 5 Crore new work opportunities and skill certifications in non-farm sector.
Education
• Mean years of schooling to increase to 7 years.
• 20 lakh seats for each age bracket in higher education.
• End gender gap and social gap in school enrollment.
Health
• Reduce : IMR to 25; MMR to 1. Increase Child Sex Ratio to 950.
• Reduce Total Fertility Rate to 2.1
• Reduce under nutrition of children in age group 0-3 to half of NFHS-3 levels.
Infrastructure
• Investment in Infrastructure at 9% of GDP
• Gross Irrigated Area 103 million hectare (from 90 million hectare)
• Electricity to all villages; Reduce AT&C losses by 20%.
• Connect Villages with All Weather Roads
• National and State high ways to a minimum of 2 lane standard.
• Complete Eastern and Western Dedicated Freight Corridors.
• Rural Tele-Density to 70%.
• 40 Litres Per Capita per Day Drinking Water to 50% of rural population; Nirmal Gram Status to
50% of all Gram Panchayats.
Environment and Sustainability
• Increase green cover by 1 million hectare every year.
• 30,000 MW renewable energy during Five Year Period.
• Emission intensity of GDP to be reduced to 20-25% of 2005 levels by 2020.
Service Delivery
• Banking Services to 90% of Indian Households.
Subsidies and Welfare related payment to be routed through Aadhar based Direct Cash Transfer
Scheme


169. With reference to the factors considered while calculating of Minimum Support Price (MSP), consider
the following statements:
1. demand and supply
2. cost of production
3. terms of trade between agriculture and non-agriculture
Select the correct answer using the code given below.
(a) 1 and 3 only (b) 2 only
(c) 2 and 3 only (d) 1, 2 and 3
Answer (d)
Explanation
In formulating the recommendations in respect of the level of minimum support prices and other non-
price measures, the Commission on Agriculture Costs and Price (CACP) takes into account the various
Terms of Reference (ToR) given to CACP in 2009. Accordingly, it analyzes :-1) demand and supply;2)
cost of production;3) price trends in the market, both domestic and international;4) inter-crop
price parity; 5) terms of trade between agriculture and non-agriculture; and 6) likely
implications of MSP on consumers of that product.
It may be noted that cost of production is an important factor that goes as an input in determination
of MSP, but it is certainly not the only factor that determines MSP.

170. With reference to the Solar Energy Corporation of India Limited (SECI), which of the following
statements is/are correct?
1. SECI is under the administrative control of the Ministry of Power
2. It was set up to facilitate the implementation of Jawaharlal Nehru National Solar Mission (JNNSM)
3. It is the only CPSU dedicated to the solar energy sector
Select the correct answer using the code given below.
(a) 1 and 2 only (b) 2 and 3 only
(c) 1 and 3 only (d) 1, 2 and 3

Answer (b)
Explanation
Solar Energy Corporation of India Ltd. (SECI) is a CPSU under the administrative control of the
Ministry of New and Renewable Energy (MNRE), set up on 20th Sept, 2011 to facilitate the
implementation of JNNSM and achievement of targets set therein. It is the only CPSU dedicated
to the solar energy sector. It was originally incorporated as a section-25 (not for-profit) company
under the Companies Act, 1956. However, through a Government of India decision, the company has
recently been converted into a Section-3 company under the Companies Act, 2013. The mandate
of the company has also been broadened to cover the entire renewable energy domain.

171. Which of the following is/are the component/components of Revenue Receipts of the Union
Government?
1. Dividends and Profits
2. Interest Receipts
3. Capital Gains Tax receipts
Select the correct answer using the code given below.
(a) 1 and 2 only (b) 2 and 3 only
(c) 1 and 3 only (d) 1, 2 and 3

Answer (d)
Explanation
Revenue Receipts includes Tax Revenue and Non-tax revenue. Capital Gains Tax is a Tax revenue
whereas Dividends & Profits and Interest Receipts are non-tax revenue.

172. Presently which of the following cesses are levied over the service tax?
1. Clean environment cess
2. Krishi Kalyan Cess
3. Swatch Bharat Cess
Select the correct answer using the code given below.
(a) 1 only (b) 1 and 3 only
(c) 2 and 3 only (d) 1, 2 and 3
Answer (c)
Explanation
In budget 2016-17 Union government introduced Krishi Kalyan cess of 0.5% over the service tax
and the amount collected used solely towards financing activities for the improvement of agriculture


and farmer welfare. It is in addition to the 0.5% of Swatch Bharat Cess already levied over the service
tax in the 2015-16 budget. Now effectively service tax is 15% (14%+0.5% Swatch Bharat Cess+ 0.5%
Krishi Kalyan Cess).
In the same budget the government doubled the clean environment cess on the coal, lignite and
peat. So it is levied on both goods as well as services.

173. Which of the following states have been awarded the Geographical Indication tag for growing Basmati
Rice?
1. Punjab
2. Delhi
3. Himachal Pradesh
4. Madhya Pradesh
Select the correct answer using the code given below.
(a) 1, 2, 3 and 4 (b) 2 and 4 only
(c) 1 and 3 only (d) 1, 2 and 3 only
Answer (d)
Explanation
Basmati rice cultivated in the Indo-Gangetic Plains on the foothills of the Himalayas covering areas of
the rice grown in Punjab, Haryana, Himachal Pradesh, Delhi, Uttarakhand, Western U.P. and two
districts of Jammu and Kathua in Jammu and Kashmir has accorded the GI tag.

174. Recently Government of India constituted Public Debt Management Cell (PDMC). Consider the
following statements regarding the PDMC:
1. It will plan Government borrowings.
2. It has only advisory functions.
3. The middle office of the Budget Division in the Finance Ministry has been subsumed into PDMC.
Which of the statements given above is/are correct?
(a) 1 only (b) 2 and 3 only
(c) 1 and 3 only (d) 1, 2 and 3
Answer (d)
Explanation
In its effort to move towards an independent Public Debt Management Agency (PDMA) in about “two
years”, the Ministry of Finance has set up a Public Debt Management Cell (PDMC) to help
streamline government borrowings and cash management for separation of debt management
functions from the Reserve Bank of India.
The PDMC, which will have only advisory functions to avoid conflict with statutory functions of the
RBI, will plan government borrowings, including market borrowings and domestic borrowing
activities like issuance of Sovereign Gold Bond. The PDMC will also manage government’s
liabilities, monitor cash balances, improve cash forecasting, foster a liquid and efficient market
for government securities along with advising government on matters related to investment,
capital market operations, administration of interest rates on small savings, among others.
PDMC will develop an Integrated Debt Database System (IDMS) as a centralised database for all
liabilities of government, on a near real-time basis and undertake requisite preparatory work for
PDMA. PDMC will have 15 experienced debt managers from Budget Division, RBI, Middle Office and
other government units.
The Middle Office of the Budget Division in the finance ministry has been subsumed into PDMC.
Joint secretary (Budget), Department of Economic Affairs will be the overall in-charge of the
PDMC, which will be located at the RBI’s Delhi office.
The government had, in the Finance Bill, 2015, proposed setting up a public debt management
agency. The aim of setting up the Public Debt Management Agency (PDMA) is to resolve issues
relating to conflict of interest as RBI decides on the key interest rates as well as undertakes
buying and selling of government bonds.

175. Inflation targeting is a monetary policy strategy used by the central banks for maintaining the prices at
a certain level or within a specific range. Consider the following statements regarding the same:
1. Central Government in consultation with RBI set the inflation target once every five years.
2. The Wholesale Price Index based inflation target of 4 percent with a band of plus/minus 2 percent
is the inflation target for RBI until March 31, 2021.
Which of the following statements given above is/are correct?
(a) 1 only (b) 2 only
(c) Both 1 and 2 (d) Neither 1 nor 2


Answer (a)
Explanation
Monetary Policy Committee uses the Consumer Price Index-(combined) based inflation target of 4
per cent with a band of plus/minus 2 percent is to be followed from 2016-17 onwards.

176. Which of the following is not a feature of both the Strategic Debt Restructuring scheme and Scheme
for Sustainable Structuring of Stressed Assets?
(a) Both schemes are unveiled by Reserve Bank of India.
(b) Both of them are aimed to meet the mounting problems of Non-Performing Assets in the banking
sector in India.
(c) Both schemes allow the lending banks to assume the ownership of the default companies.
(d) None of the above.
Answer (c)
Explanation
The RBI came up with the SDR scheme in June 2015 to provide an opportunity to banks to convert
debt of companies (whose stressed assets were restructured but which could not finally fulfil the
conditions attached to such restructuring) to 51 percent equity and sell them to the highest bidders,
subject to authorization by existing shareholders. An 18-month period was envisaged for these
transactions, during which the loans could be classified as performing. But as of end-December 2016,
only two sales had materialized, in part because many firms remained financially unviable, since only
a small portion of their debt had been converted to equity.
The S4A is RBI’s latest attempt to try and chip away at the mountain of stressed loans smothering
Indian banks. The scheme tries to strike a compromise between banks with problem accounts and
corporate defaulters of the non-wilful variety, by converting a portion of large loan accounts
into equity shares.
For a distressed company to be eligible for S4A, the RBI has laid down three conditions.
1. The project must be operating and already generating cash.
2. The total loans to the entity should be ₹500 crore or more.
3. The lending banks are required to hire an independent agency to evaluate how much of the debt is
‘sustainable’. For the loan to be eligible for S4A, at least 50 per cent of it should be ‘sustainable’.
While RBI’s earlier ideas to resolve bad loans such as Strategic Debt Restructuring (SDR) required
banks to oust existing promoters, S4A allows the incumbent management to continue, as long the
default isn’t wilful.

177. Government of India unveiled the ‘Income Declaration Scheme’ (IDS), 2016 for those who hold black
money to come clean by paying taxes under the scheme. Consider the following statements regarding
the same:
1. Under this scheme, persons can declare the undisclosed income and pay tax, surcharge and
penalty on the declared undisclosed income.
2. The person making a declaration under the scheme will have to pay income tax at the rate of 30%
of the value of undisclosed income plus a surcharge of 25% thereon.
3. Additionally, the person will be liable to pay a penalty at the rate of 25% of the tax, which would
make the total payment 45% of the undisclosed income.
Which of the statements given above is/are correct?
(a) 1 only (b) 2 only
(c) 1 and 3 only (d) 1, 2 and 3
Answer (d)
Explanation
1. Income Declaration Scheme 2016 came into effect from 1 June, 2016. It's an opportunity for those
who didn't disclose income or pay taxes thereon in the past.
2. Under this scheme, such persons can declare the undisclosed income and pay tax, surcharge and
penalty on the declared undisclosed income.
3. Declaration of undisclosed income or asset should have been chargeable to tax for any assessment
year prior to the assessment year 2017-18.
4. The person making a declaration under the scheme will have to pay income tax at the rate of 30%
of the value of undisclosed income plus a surcharge of 25% thereon.
5. Additionally, the person will be liable to pay a penalty at the rate of 25% of the tax, which would
make the total payment 45% of the undisclosed income.
6. The Centre has declared 30 September as the last date for making a declaration and 30 November
as the date by which the tax, surcharge and penalty may be paid.


178. The Department of Disinvestment has been renamed as Department of Investment and Public Asset
Management (DIPAM) from 14th April, 2016. Which of the following statements regarding the DIPAM
is/are correct?
1. DIPAM will create a financial policy in regard to the utilisation of the proceeds of disinvestment
channelised into the National Investment Fund.
2. DIPAM is mandated to advice and identify the CPSEs for strategic sale.
Which of the statements given above is/are correct?
(a) 1 only (b) 2 only
(c) Both 1 and 2 (d) Neither 1 nor 2
Answer (d)
Explanation
The Department of Disinvestment was set up as a separate Department on 10th December, 1999 and
was later renamed as Ministry of Disinvestment form 6th September, 2001.
From 27th May, 2004, the Department of Disinvestment is one of the Departments under the Ministry
of Finance.
The Department of Disinvestment has been renamed as Department of Investment and Public Asset
Management (DIPAM) from 14th April, 2016.
Ahead of Union Budget for 2017-18, the Department of Investment and Public Asset Management
(DIPAM) stripped off its mandate to frame policy on the utilisation of disinvestment proceeds.
Instead, the Department of Economic Affairs in the Finance Ministry will now create “financial policy
in regard to the utilisation of the proceeds of disinvestment channelised into the National Investment
Fund,” as per a notification dated January 16, changing the Allocation of Business, signed by
President Pranab Mukherjee.
In addition, the role of DIPAM in strategic sales was handed over to the core group of secretaries on
disinvestment headed by the Cabinet Secretary. For the first time, NITI Aayog was mandated to
advice and identify the CPSEs for strategic sale to the core group of secretaries.
The Centre had created a National Investment Fund in 2005 into which the proceeds from
disinvestment of Central Public Sector Enterprises were to be channelised. CPSEs are firms where
the direct holding of the government or other CPSEs is at least 51%.

179. Consider the following statements with regard to Pradhan Mantri Ujjwala Yojana:
1. It aims to provide Free LPG connections to all women households across the country.
2. It aims to reduce the serious health hazards associated with cooking based on fossil fuel.
3. The scheme will be implemented by the Ministry of Petroleum & Natural Gas.
Which of the following statements given above are correct?
(a) 1 and 2 only (b) 2 and 3 only
(c) 1 and 3 only (d) 1, 2 and 3
Answer (b)
Explanation
The Pradhan Mantri Ujjwala Yojan was launched on 1st May, 2016. It is aimed at providing 5 Crore
LPG connections in the name of women in BPL (Below Poverty Line) households across the country.
It aims to reduce the serious health hazards associated with cooking based on fossil fuel. The
scheme is also for reducing the number of deaths in India due to unclean cooking fuel. The scheme
will be implemented by the Ministry of Petroleum & Natural Gas (first time in the history). This
Scheme would be implemented over three years, namely, the FY 2016-17, 2017-18 and 2018-19.

180. ‘Pradhan Mantri Awaas Yojana’ has been launched for


(a) creating efficient and effective institutional platforms of the rural poor
(b) provide all-weather road connectivity to all eligible unconnected habitations
(c) building affordable pucca houses with water facility, sanitation and electricity supply
(d) transforming urban living conditions through infrastructure upgradation
Answer (c)
Explanation
Pradhan Mantri Awas Yojana (PMAY- Housing for all) was launched in June 2015. The Government
envisages building affordable pucca houses with water facility, sanitation and electricity supply round-
the-clock. Option (a) is the objective of Deendayal Antyodaya Yojana – National Rural Livelihoods
Mission (DAY-NRLM). Option (b) is the objective of Pradhan Mantri Gram Sadak Yojana (PMGSY).
Option (d) is the purpose of Atal Mission for Rejuvenation and Urban Transformation (AMRUT)
scheme.

181. Which of the following is/are example(s) of ‘Near Money’?


1. Treasury Bill


2. Savings accounts and small time deposits
3. Certificates of Deposit
Select the correct answer using the code given below:
(a) 1 and 2 only (b) 1 and 3 only
(c) 2 and 3 only (d) 1, 2 and 3

Answer (d)
Explanation
Near money is an economics term describing non-cash assets that are highly liquid, such as bank
deposits, certificates of deposit (CDs) and Treasury Bills. Near money refers to assets that can be
quickly converted into cash. Also called quasi-money.

182. In the context of which of the following, do you sometimes find the term ‘Advanced Pricing Agreement’?
(a) Tax matters
(b) Carbon Trading
(c) Solar power generation
(d) Wind power generation
Answer (a)
Explanation
The APA Scheme was introduced in the Income-tax Act in 2012 and the “Rollback” provisions were
introduced in 2014. The scheme endeavours to provide certainty to taxpayers in the domain of transfer
pricing by specifying the methods of pricing and setting the prices of international transactions in
advance. Since its inception, the APA scheme has evinced a lot of interest from taxpayers and that has
resulted in more than 700 applications (both unilateral and bilateral) being filed so far in about five
years.
The progress of the APA Scheme strengthens the Government’s resolve of fostering a non-adversarial
tax regime. The Indian APA programme has been appreciated nationally and internationally for being
able to address complex transfer pricing issues in a fair and transparent manner.

183. Consider the following statements regarding the FDI in multi brand retail:
1. 51% FDI is allowed in multi brand retail under automatic route.
2. 100% FDI is allowed in multi brand food retail stores via approval route.
Which of the statements given above is/are correct?
(a) 1 only (b) 2 only
(c) Both 1 and 2 (d) Neither 1 nor 2
Answer (b)
Explanation
51% FDI in multi brand retail is done under the approval route.

184. Which of the following statements is correct with respect to the composition of national income in
India?
1. The share of manufacturing sector has increased
2. The share of services sector has increased sharply.
3. The share of agriculture has remained static
4. The share of agriculture has declined
Which of the above statements are correct?
(a) 1, 2 and 3 (b) 1, 2 and 4
(c) 2, 3 and 4 (d) 1, 3 and 4
Answer (b)
Explanation
With respect to National Income in India, these are the changes in the respective sector
1. The share of manufacturing sector has increased
2. The share of services sector has increased sharply.
3. The share of agriculture has declined

185. Consider the following:


1. Currency with the public
2. Demand deposits of the banks
3. Time deposits of the banks
4. Other deposits of RBI
Which of the following are included in the Narrow Money in India?
(a) 1 and 2 (b) 1, 2 and 3
(c) 1, 2, 3 and 4 (d) 1, 2 and 4


Answer (d)
Explanation
Narrow Money= Currency with the public+ Demand deposits of the banks+ Other deposits of RBI

186. ‘Peer to Peer lending’ sometimes seen in the news is related to


(a) Global financial crisis, 2008
(b) Mutual finance assistance by developing countries
(c) Unsecured lending via online platforms
(d) Double non-taxation

Answer (c)
Explanation
Peer-to-peer lending refers to unsecured lending that happens on online platforms without the
involvement of a bank or a finance company. A P2P lender simply brings lenders and borrowers
together on a common online platform so that they can transact with each other.

187. Board for Industrial and Financial Reconstruction is under


(a) Ministry of Heavy Industries and Public Enterprises
(b) Reserve Bank of India
(c) Ministry of Commerce and Industry
(d) None of the above

Answer (d)
Explanation
The Board for Industrial and Financial Reconstruction (BIFR) was an agency of the government
of India, part of the Department of Financial Services of the Ministry of Finance. The BIFR was
established under The Sick Industrial Companies (Special Provisions) Act, 1985 (SICA).
Its objective is to determine sickness of industrial companies and to assist in reviving those that
may be viable and shutting down the others. But from 1 December 2016, by an official notification,
Government of India dissolved it and all proceedings to be referred to the National Company Law
Tribunal (NCLT) and National Company Law Appellate Tribunal (NCLAT) as per provisions
of Insolvency and Bankruptcy Code.

188. Consider the following statements regarding the Real Estate Investment Trusts (REITs):
1. Real Estate Investment Trust refers to an entity created with the sole purpose of channeling
investible funds into operating, owning or financing income-producing real estate.
2. REITs are modelled on the lines of mutual funds and provide investors with an extremely liquid
way to get a stake in real estate.
3. REITs can enlist themselves on a stock exchange.
Which of statements given above is/are correct?
(a) 1 and 2 only (b) 2 and 3only
(c) 1 and 3 only (d) 1, 2 and 3
Answer (d)
Explanation
1. Real Estate Investment Trust refers to an entity created with the sole purpose of channeling
investible funds into operating, owning or financing income-producing real estate.
2. REITs are modelled on the lines of mutual funds and provide investors with an extremely liquid
way to get a stake in real estate.
3. REITs can enlist themselves on a stock exchange.
In India, the Real Estate Investment Trusts were introduced by the Securities and Exchange Board
of India (Sebi) in 2007. The securities watchdog only released draft regulations which due to
certain limitations were later on rejected. In September, 2013, Sebi came out with revised
regulations for REITs, which were approved on September 26, 2014.
REITs have many advantages for interested investors. It provides a regular income stream along
with reduced portfolio volatility and dividends and wealth accumulation. As a result of it being a
listed entity, it is bought and sold with ease providing great liquidity. It is a natural hedge against
inflation as returns have been seen to consistently outpace Consumer Price Inflation.
There are primarily two types of REITs - equity and mortgage. Real Estate Investment Trusts are
extremely beneficial for the development of an economy as they allow dormant investable funds to
be channelled into infrastructure projects such as apartment complexes, hospitals, schools and the
likes.

189. The 7.1% growth rate registered by Indian economy during the year 2016-17 is based on
(a) Gross National Product at market prices


(b) Gross Value Added at constant prices
(c) Gross Domestic Product at market prices
(d) Gross Domestic Product at constant prices
Answer (c)
Explanation
The Government’s statistics wing made two changes to the GDP calculation in January 2015. One
was a change in the base year for the calculation which is done routinely every five years or so. The
other was to adopt a new method to measure output. Starting now, Indian GDP will be measured by
using gross domestic product (GDP) at market price, rather than factor cost.
As for the base year change, it is the only way to ensure that the products and services included in
the GDP calculation do remain contemporary and reflect the present state of the economy. For
instance, the latest change in base year from 2004-05 to 2011-12 has included the recycling
industry which didn’t figure in the earlier GDP computations.
Similarly trading activities by manufacturing firms are now included in that sector’s share. This
change along with better data compilation (online data filed with the Ministry of Corporate Affairs)
has led to manufacturing increasing its share in GDP.

190. With reference to the Khadi and Village Industries Commission (KVIC), which of the following
statements is/are correct?
1. The Khadi and Village Industries Commission (KVIC) is a statutory body.
2. It has the social objective of providing employment and the economic objective of producing
saleable articles.
3. It is aimed at creating self-reliance amongst the poor and building up of a strong rural community
spirit.
Select the correct answer using the code given below.
(a) 1 and 3 only (b) 2 only
(c) 2 and 3 only (d) 1, 2 and 3
Answer (d)
Explanation
The Khadi and Village Industries Commission (KVIC) is a statutory body established by an act of
parliament. In April 1957, it took over the work of former All India Khadi and Village Industries Board.
The broad objectives that the KVIC has set before it are...
o The social objective of providing employment.
o The economic objective of producing saleable articles.
o The wider objective of creating self-reliance amongst the poor and building up of a strong
rural community spirit.
191. With reference to ‘Interest subvention scheme for farmers’, consider the following statements:
1. The interest subvention scheme for farmers aims at providing long term credit to farmers at
subsidised interest rate.
2. This scheme covers post-harvest loans and relief to farmers affected by natural calamities.
Which of the statements given above is/are correct?
(a) 1 only (b) 2 only
(c) Both 1 and 2 (d) Neither 1 nor 2
Answer (b)
Explanation
The interest subvention scheme for farmers aims at providing short term credit to farmers at
subsidised interest rate. The policy came into force with effect from Kharif 2006-07. The scheme is
being implemented for the year 2016-17.
➢ Interest subvention for short term crop loans
➢ The Central Government provide farmers under the interest subvention scheme to all farmers for
short term crop loan upto one year for loan upto Rs. 3 lakhs borrowed by them during the
year 2016-17.
➢ Farmers will thus have to effectively pay only 4% as interest. In case farmers do not repay the
short term crop loan in time they would be eligible for interest subvention of 2% as against 5%
available above.
➢ The amount of subvention is to be calculated on the amount of crop loan from the date of
disbursement up to the date of repayment.
➢ Interest subvention for post harvest loans
In order to give relief to small and marginal farmers who would have to borrow at 9% for the post
harvest storage of their produce, the Central Government has approved an interest subvention of
2% i.e an effective interest rate of 7% for loans upto 6 months.


➢ Interest subvention for relief to farmers affected by natural calamities
To provide relief to the farmers affected by Natural Calamities, the interest subvention of 2% will be
provided to Banks for the first year on the restructured amount.
➢ Due to the cancellation of legal tender character of old Rs. 500 and Rs.1000 notes and the
resulting difficulty faced by the farmers, an interest waiver was provided for a period of 2
months i.e. November & December 2016 to farmers who were disbursed crop loan from
Cooperative Banks between 01.04.2016 and 30.09.2016.

192. In the context of which of the following do you sometimes find the term ‘anti profiteering clause’?
(a) Goods and Service Tax
(b) Base Erosion and Profit Shifting
(c) Regional Comprehensive Economic Partnership
(d) India-Mauritius DTAA
Answer (a)
Explanation
The GST law contains an anti-profiteering clause that mandates a manufacturer and others in the
supply chain to pass on the benefits arising out of input credit and lower taxes to consumers at the
pain of penalty.
As per the Central GST Act, the Central Government may by law set up an authority or designate an
existing authority to examine if reduction in tax rate has resulted in commensurate reduction in prices
of goods and services. The powers of the authority will be prescribed by the Government.

193. Goods and Services Tax (GST) is the one of the major tax reforms in the indirect tax system of country.
Which of the following statements is/are correct regarding the same?
1. The GST is expected to roll out from the beginning of the financial year 2017-18.
2. The Central GST Bill provides for levy and collection of tax on intra-State supply of goods or
services or both by the Central government.
3. The Integrated GST Bill makes provisions for levy and collection of tax on inter-State supply of
goods or services or both by the Central government.
Select the correct answer using the code given below.
(a) 1 and 2 only (b) 2 and 3 only
(c) 1 and 3 only (d) 1, 2 and 3
Answer (b)
Explanation
The GST is expected to be rolled out from the 1st of July 2017. The four bills viz. Central GST bill,
UT GST bill, Integrated GST bill and Compensation Bill have been recently passed by the
Parliament. The State GST Bill will be taken up by State Cabinets and introduced in each State
Assembly.
The UTGST Bill makes provisions for levy on collection of tax on intra-UT supply of goods and
services in Union Territories without a legislature. The UTGST is akin to the States Goods and
Services Tax (SGST), which will be levied and collected by the States/Union Territories with
legislature on intra-State supply of goods or services or both.

194. Consider the following taxes:


1. Service Tax
2. Capital Gains Tax
3. Securities Transaction Tax
Which one of the above is/are Indirect Tax/Taxes?
(a) 1 only (b) 1 and 3 only
(c) 2 and 3 only (d) 1, 2 and 3
Answer (a)
Explanation
Those taxes for which, the burden of the tax falls on the entity that is being taxed are known as direct
taxes. Eg: Corporation Tax, Taxes on Personal Income, Securities Transaction Tax, Capital Gains Tax,
Wealth Tax, Interest |Tax etc.
Those taxes for which, the burden of the tax falls on the entity that is being taxed are known as direct
taxes. Rg: Corporation Tax, Taxes on Personal Income, Securities Transaction Tax, Capital Gains Tax,
Wealth Tax, Interest Tax etc.
Those taxes for which the tax burden can be shifted or passed on to other persons later through
business transactions of goods / services are called the Indirect taxes. These taxes are indirect
because the agent who bears the burden of the tax is not the one on whom it is normally levied. Eg:
Service Tax, GST, VAT, Customs Duties, Sales Tax, Entry Tax etc.


195. In economy, the term for establishments having formal existence but with close to no operations or
assets is termed as
(a) tax haven
(b) shell company
(c) benami property
(d) sick industry
Answer (b)
Explanation
A shell corporation is a corporation without active business operations or significant assets.
These types of corporations are not all necessarily illegal, but they are sometimes used
illegitimately, such as to disguise business ownership from law enforcement or the public. Shell
corporations act as tax avoidance vehicles for legitimate businesses.

196. Which of the following constitute Foreign Exchange Reserves of India?


1. Foreign currency assets
2. Gold holdings of RBI
3. Grants from international institutions
4. Special Drawing Rights
Select the correct answer using the code given below.
(a) 1, 2 and 3 only (b) 1, 2 and 4 only
(c) 2, 3 and 4 only (d) 1, 3 and 4 only
Answer (b)
Explanation
Foreign Exchange Reserve of India constitutes foreign currency assets, gold stocks of RBI, Special
Drawing Rights holdings of the Government and Reserve Tranche position in IMF.

197. The proposed ‘Payment Regulatory Board’ in Budget 2017-18 is


(a) an independent body to bring about structural reforms in the payment eco-system.
(b) headed by the Reserve Bank Governor.
(c) constituted by the members of the RBI only.
(d) recommended by NK Singh Committee.
Answer (b)
Explanation
Finance Minister Arun Jaitley proposed creation of a six-member Payments Regulatory Board in the
RBI, headed by its Governor, as part of bringing about structural reforms in the payment eco-system
in the Budget speech of 2017-18. It will replace the existing Board for Regulation and Supervision of
Payment and Settlement Systems.
The RBI Governor will be Chairperson, ex officio. The Deputy Governor who is in—charge of the
Payment and Settlement Systems—Member, and one officer of the Reserve Bank to be nominated by
its Central Board will be its members. The central government will also nominate three members to the
Board.
It was recommended by Ratan Watal committee on digital transactions.

198. Which of the following oil companies are proposed to merge and create an integrated oil major in the
public sector in the Budget 2017-18?
1. Bharat Petroleum Corporation of India
2. Hindustan Petroleum Corporation of India
3. Oil and Natural Gas Corporation of India
4. Essar Oil
Select the correct answers using the code given below.
(a) 1, 2 and 3 only (b) 1, 2 and 4 only
(c) 2, 3 and 4 only (d) 1, 3 and 4 only
Answer (a)
Explanation
In the budget speech of 2017-18, the Finance Minister of India proposed to create an integrated
public sector ‘oil major’ which will be able to match the performance of international and domestic
private sector oil and gas companies.
State-owned firms in the oil and gas sector are: Oil and Natural Gas Corp. Ltd (ONGC), Oil India Ltd,
GAIL (India) Ltd, Indian Oil Corp. Ltd, Bharat Petroleum Corp. Ltd, Hindustan Petroleum Corp.
Ltd, Numaligarh Refinery Ltd, Chennai Petroleum Corp. Ltd, Engineers India Ltd, Balmer Lawrie
and Co. Ltd, and Biecco Lawrie Co. Ltd.


The major private players in oil sector of India are Reliance Industries, Shell, Essar oil etc.

199. Commodity Future trading in India is regulated by


(a) Forward Market Commission
(b) Reserve Bank of India
(c) Securities and Exchange Board of India
(d) Union Government of India
Answer (c)
Explanation
Earlier Commodity Future Market is regulated by Forward Market Commission (FMC). But in 2015
budget speech, the Finance Minister has declared the merger of FMC with Securities and Exchange
Board of India (SEBI) and brought the Commodity Trading under the latter’s purview.

200. Which of the following countries are part of both Trans Pacific Partnership (TPP) and North American
Free Trade Agreement (NAFTA)?
1. Canada
2. Peru
3. Mexico
4. Chile
Select the correct answer using the code given below.
(a) 1, 2 and 3 only (b) 2 and 4 only
(c) 1 and 3 only (d) 1, 2, 3 and 4

Answer (c)
Explanation
Twelve countries that border the Pacific Ocean signed up to the Trans Pacific Partnership (TPP) trade
agreement in February 2016, representing roughly 40% of the world's economic output. These are
Canada, USA, Mexico, Peru, Chile, New Zealand, Australia, Vietnam, Philippines, Brunei,
Singapore and Japan.
Recently new President of USA, Donald Trump has announced the withdrawal of USA from the pact.
North American Free Trade Agreement (NAFTA) is the trade agreement among Canada, USA and
Mexico.


You might also like